You are on page 1of 103

30 Clinical caasses

From

By: Dr. Ali Faris Haider

23/11/2008

This Book is not published by eMedicine .It is just a selection of cases from eMedicine website by Ali Faris
Haider.

3o clinical cases from eMedicine

Preface
Since the best way of learning medicine is to study medical problems
I collected some of the common cases from eMedicine website and put them in this book without
editing the contents so that everyone can enjoy solving these Medical problems.
What is eMedicine?
eMedicine: It is web-based and consists of clinical overviews of disease entities by experts in the
field. Continuously updated, mainly for professionals, over 10 000 physician authors on 7000
diseases, Articles undergoes 4 levels of physician peer review plus an addi onal review by a
Pharmacy editor prior to publication, 30 000 mul media les,12 % of radiology residents use it as
the first source of information.
eMedicine is read by doctors and medical students from approximately 120 countries.
eMedicine(www.emedicine.com) sends a weekly case via email to its subscribers.
This book is dedicated to Junior and senior doctors.
If you find this book useful; please remember me in your prayer.
Dr. Ali Faris Haider
13-12-2008

3o clinical cases from eMedicine


Contents
Case 1......................................................................................................................................................................... 3
Case 1 Answer............................................................................................................................................................. 5
Case 2......................................................................................................................................................................... 7
Case 2 Answer............................................................................................................................................................. 8
Case 3......................................................................................................................................................................... 9
Case 3 Answer........................................................................................................................................................... 11
Case 4....................................................................................................................................................................... 12
Case 4 Answer........................................................................................................................................................... 14
Case 5....................................................................................................................................................................... 17
Case 5 Answer........................................................................................................................................................... 18
Case 6....................................................................................................................................................................... 19
Case 6 Answer........................................................................................................................................................... 22
Case 7....................................................................................................................................................................... 24
Case 7 Answer........................................................................................................................................................... 25
Case 8....................................................................................................................................................................... 26
Case 8 Answer........................................................................................................................................................... 29
Case 9....................................................................................................................................................................... 33
Case 9 Answer........................................................................................................................................................... 35
Case 10 ..................................................................................................................................................................... 39
Case 10 Answer ......................................................................................................................................................... 40
Case 11 ..................................................................................................................................................................... 43
Case 11 Answer ......................................................................................................................................................... 44
Case 12 ..................................................................................................................................................................... 45
Case 12 Answer ......................................................................................................................................................... 47
Case 13 ..................................................................................................................................................................... 49
Case 13 Answer ......................................................................................................................................................... 50
Case 14 ..................................................................................................................................................................... 51
Case 14 Answer ......................................................................................................................................................... 52
Case 15 ..................................................................................................................................................................... 53
Case 15 Answer ......................................................................................................................................................... 54
Case 16 ..................................................................................................................................................................... 55
Case 16 Answer ......................................................................................................................................................... 56
Case 17 ..................................................................................................................................................................... 57
Case 17 Answer ......................................................................................................................................................... 58
Case 18 ..................................................................................................................................................................... 59
Case 18 Answer ......................................................................................................................................................... 60
Case 19 ..................................................................................................................................................................... 61
Case 19 Answer ......................................................................................................................................................... 62
Case 20 ..................................................................................................................................................................... 63
Case 20 Answer ......................................................................................................................................................... 64
Case 21 ..................................................................................................................................................................... 65
Case 21 Answer ......................................................................................................................................................... 66
Case 22 ..................................................................................................................................................................... 67
Case 22 Answer ......................................................................................................................................................... 68
Case 23 ..................................................................................................................................................................... 69
Case 23 Answer ......................................................................................................................................................... 72
Case 24 ..................................................................................................................................................................... 76
Case 24 Answer ......................................................................................................................................................... 78
Case 25 ..................................................................................................................................................................... 80
Case 25 Answer ......................................................................................................................................................... 82
Case 26 ..................................................................................................................................................................... 84
Case 26 Answer ......................................................................................................................................................... 86
Case 27 ..................................................................................................................................................................... 88
Case 27 Answer ......................................................................................................................................................... 90
Case 28 ..................................................................................................................................................................... 92
Case 28 Answer ......................................................................................................................................................... 94
Case 29 ..................................................................................................................................................................... 97
Case 29 Answer ......................................................................................................................................................... 99
Case 30 ....................................................................................................................................................................101
Case 30 Answer ........................................................................................................................................................102

3o clinical cases from eMedicine


Case 1
RAPID HEART RATE AND SHORTNESS OF BREATH IN A 40-YEAR-OLD MAN
Background
A 40-year-old man presents to the emergency department (ED) with palpitations and shortness of breath that
started a few minutes before his arrival.
The patient states that he was closing his shop when his heart began to beat rapidly and he had difficulty
catching his breath. His symptoms started suddenly and continued when paramedics arrived minutes later.
They observed a rapid heart rate on the cardiac monitor and associated rhythm strips (see Figures 1-2). He was
subsequently given adenosine 6 mg en route to the hospital. The pa ent had a momentary period of asystole,
but his rapid heart rate returned.

Figure 1

Figure 2

On his arrival to the emergency department, the patient continued to have the sensation that his heart was
racing. He denies having any chest pain, nausea, vomiting, diaphoresis, light-headedness, or recent illness. He
felt well before this episode. He denies having any symptoms of infection, such as fever, cough, vomiting,
diarrhea, anorexia, or dysuria. He reports increased stress at work and is drinking as many as 4 cups of coee a
day. He reports no notable history of medical conditions except for a similar episode of a rapid heart rate about
4 years ago; for this, he was treated with an unknown drug for 2 years. His family history is signicant for a
father who died of a myocardial infarc on at 45 years of age. The pa ent takes 1 baby aspirin daily. He denies
using any over-the-counter or illicit drugs; however, he smokes 3 packs of cigare es per week.
On physical examina on, the pa ent is afebrile and has a heart rate of 165 bpm and a blood pressure of 138/79
mm Hg. He appears well and is in no acute distress. Findings on head and neck examination are unremarkable.
He has no jugular venous disten on. His heart rate is rapid and irregular, with an audible S1 and S2 and no
gallops, rubs, or murmurs. His lungs are clear bilaterally. His abdomen is soft, nontender, and without any

3o clinical cases from eMedicine


masses. He has no peripheral edema. Results of his laboratory workup, including a CBC, serum electrolyte and
cardiac enzyme measurements, and a coagulation panel, are all normal. His chest radiograph is also normal.

An ECG is obtained (see Figure 3).

Figure 3

What is the diagnosis?


Hint: Paramedics gave the patient a single dose of adenosine, which had no lasting effect on the rapid rhythm.
o Ventricular fibrillation
o Atrial fibrillation
o Torsade de pointes
o Sinus tachycardia

3oo clinical cases from eMedicine


Case 1 Answer
RAPID HEART R ATE AND SHORTNESS OF BREATH IN A 40-YEAR -OLD MAN

Diagnosis:: Atrial fibrillation (AF) with a rapid ventricular response.


Discussion:
The ECG shows an irregular heart rhythm with no discernible P waves. Rapid atrial fibrillation (AF) may be hard
to differentiate from a narrow supraventricular tachycardia (SVT) without close examination of an ECG. The two
conditions can result in similar symptoms of heart palpitations
palpitations and shortness of breath. Patients with rapid AF
are not uncommonly given adenosine to treat presumed SVT, as in this case. Although this treatment is typically
unsuccessful, the underlying atrial rhythm may be accurately determined when the heart rate briefly slows.
The conversion from a normal sinus rhythm to AF may be due to a number of conditions, including
hyperthyroidism, anemia, infection, ischemic heart disease, valvular disease, drug intoxication, or use of
stimulants. Increased stress and overconsumption
overconsumption of coffee are likely to have been the instigating factors in this
patient.
AF is a common arrhythmia characterized by chaotic atrial depolarizations without effective atrial contractions.
This rhythm is often seen with increasing age, with a male
male predominance. This arrhythmia can result in
decreased cardiac output and the formation of atrial thrombi. Many patients with AF are asymptomatic, and
most have recurrent episodes without knowledge of them.
The American College of Cardiology established a classification system for AF that is based on its duration and
etiology. The categories are paroxysmal AF, persistent AF, permanent AF, and lone AF. In paroxysmal AF, the
episodes last less than 1 week. If they recur, the condi on is considered recurrent paroxysmal AF. In persistent
AF, the episodes last longer than 1 week. In permanent AF, the episode lasts longer than 1 year without any
attempts for conversion or with attempts that fail. Finally, in lone AF, no underlying structural cardiac or
pulmonaryy disease is found. Patients with lone AF have a low risk of mortality and thromboembolism and may
have paroxysmal, persistent, or permanent AF.
The workup for AF involves careful history taking and physical examination, laboratory studies (including a CBC
CBC,
serum electrolyte tests, toxicology screening, and thyroid function tests), ECG, chest radiography, and
echocardiography.
The patient's history should include the time of onset, the frequency of episodes, any associated symptoms, and
any history of treatment
ment for AF. Laboratory studies may be useful in determining possible etiologies of AF. The
WBC count may help in finding an underlying infection, and the hemoglobin concentration may demonstrate
anemia. Electrolyte levels, such as magnesium and potassium levels, may be abnormal, and an elevated
creatinine value may indicate a renal insufficiency. Certain illicit drugs can cause a rapid heart rate; therefore, a
toxicology screening may be useful when indicated. Hyperthyroidism can predispose patients to AF. For this
reason, an evaluation of thyroid function with measurement of the patient's thyroid
thyroid-stimulating hormone (TSH)
level is warranted.
AF can be diagnosed when the ECG shows an irregular rhythm with the absence of P waves. In addition,
examine the patient
ient for any signs of left ventricular hypertrophy, bundle branch blocks, and atrioventricular

3o clinical cases from eMedicine


(AV) nodal blocks, as well as for evidence of cardiac ischemia or previous myocardial infarction. Chest
radiographs may be useful in evaluating the cardiac silhouette for cardiomegaly and the lung fields and
vasculature for evidence of airspace disease or pulmonary edema. A transthoracic echocardiogram can be
obtained to identify the size and motion of the atria, ventricles, and cardiac valves, and it can reveal pericardial
disease. Transesophageal echocardiography is more sensitive than transthoracic echocardiography for
diagnosing left atrial thrombus or left atrial appendage thrombus.
Rate control is important in pa ents who present with rapid AF of more than 72 hours' dura on, and beta
blockers (metoprolol or atenolol) or calcium channel blockers (verapamil or diltiazem) are recommended in
patients who do not have an accessory pathway. Digoxin and amiodarone are the drugs of choice for controlling
rapid AF in patients with left ventricular failure and no accessory pathway; however, digoxin should be loaded
over 24 hours. Therefore, it is unlikely to have a notable eect in the acute se ng. If unable to achieve rate
control with pharmacologic therapy, catheter-directed AV nodal ablation by a cardiologist may be considered.
Anticoagulation treatment is also recommended for most patients with persistent AF, and it is typically achieved
with warfarin (dosed to maintain an interna onal normalized ra o [INR] of 2.0
-3.0). In pa ents considered to be
at low risk for thromboembolism or in patients who have a contraindication to the use of warfarin, aspirin can
be given instead.
Conversion to a sinus rhythm may be achieved with pharmacologic agents or with synchronized external
electrical cardioversion. Conversion should be done only when the risk of thromboembolism is limited, as in
pa ents with an onset of symptoms less than 72 hours before presenta on, in those who received
e in whom transesophageal echocardiographic results rule out a left atrial
an coagula on for 3 weeks, or in thos
thrombus.
A er successful cardioversion, an coagula on therapy should con nue for at least 1 month to decrease the risk
of thromboembolism, which may occur from the formation of a mural thrombus. After cardioversion is done
and the patient's AF reverts to a sinus rhythm, use of daily outpatient antiarrhythmic drugs is not typically
recommended, as these drugs have associated risks; therefore, they should be taken only when patients have
persistent or frequently recurring symptoms.
Antiarrhythmic drugs that can be used to convert AF to a normal sinus rhythm include ibutilide, flecainide,
procainamide, and amiodarone. Each has different risks, success rates, and indications based on the duration of
AF. As a group, an arrhythmic drugs can convert 30-60% of cases of AF to a normal sinus rhythm. Electrical
cardioversion has a higher success rate, conver ng 75-95% of AFs to normal sinus rhythms. Electrical
cardioversion may be done in a nonemergency se ng a er 3 weeks of an coagula on treatment to decrease
the risk of thromboembolism, or it may be required on an emergency basis in a hemodynamically unstable
patient. In this situation, AF often has an acute onset, and the benefits of cardioversion outweigh the risks of
thromboembolism.
The role of cardioversion to manage AF in the emergency department is an emerging one. Patients who are at
low risk, who are clinically stable, and who present to the emergency department with new-onset AF can be
treated with chemical or electrical cardioversion and safely discharged, home with close follow-up, by a primary
physician or cardiologist.

3o clinical cases from eMedicine


Case 2
CHEST PAIN FOR 2 DAYS
Background
A 38-year-old man without cardiac risk factors presents with chest pain that has been con nuous for 2 days. His
vital signs are as follows: temperature, 99F; heart rate, 86 beats per minute; blood pressure, 140/78 mm Hg;
and respiratory rate, 21 per minute .

Figure 1

Hint
The pa

ent looks comfortable despite changes on his ECG. He had symptoms of a common cold 3 weeks ago.

3o clinical cases from eMedicine


Case 2 Answer
CHEST PAIN FOR 2 DAYS
Diagnosis: Acute pericarditis
Discussion:
The ECG shows a normal sinus rhythm (NSR) that is regular, with normal intervals. ST elevation is notable in
leads II, III, aVF, and V3 through V6. Note that the PR depression is most visible in leads II and V2 through V6.
Corresponding PR elevation is noted in the aVR lead .
The ECG findings are consistent with acute pericarditis, which can cause friction rub to be heard on
examina on. In stage I (as shown here), diuse ST eleva on is present in leads I, II, aVL, aVF, and V3 through V6,
and reciprocal ST depressions are seen in leads aVR and V1. Inamma on cannot occur supercial to the thinwalled atria; therefore, some myocarditis occurs. Exaggerated atrial T waves cause PR-segment depressions
with a polarity opposite to that of the P wave. PR elevation, as shown in the aVR lead, is a reliable indicator of
pericarditis. In stage II, the ST elevations and PR depressions resolve, and the ECG normalizes. In stage III,
generalized T-wave inversions are present in most or all leads. In stage IV, the ECG returns to its state before
the pericarditis, or the T-wave inversions become permanent.

3o clinical cases from eMedicine


Case 3
FEBRILE PNEUMONIA IN A MEDICAL STUDENT RETURNING FROM SOUTHEAST ASIA
Background
A 25-year-old fourth-year Filipino medical student presents to a California walk-in clinic with an intermittent
fever, chills, and cough las ng almost 5 days. He has had progressively worsening muscle aches. Over the last 48
hours, he has noticed increasing difficulty in taking a deep breath. Because the patient has had no prior medical
illnesses and only occasional colds, he did not seek medical care earlier .

Figure 1

For the rst 2 days of this illness, the pa ent had a mild runny nose, a sore throat, and some diarrhea, all of
which were self-limited. Since his illness began, he has had an on-and-off headache, in addition to increasing
weakness and anorexia. On systemic review, the findings are otherwise essentially negative, including the

3o clinical cases from eMedicine


absence of a rash, headache, abdominal pain, vomiting, and back pain. The patient smokes less than a pack of
cigarettes per day but does not have chronic bronchitis. He denies using alcohol, illicit drugs, and prescription
medications .
The pa ent is nishing an 8-week hospital-based rotation in Southeast Asia in which he was an acting intern. He
wanted to study internal medicine and critical care medicine in the region because he aspires to work in
interna onal health. During his rota on, he was in Manila for 4 weeks and then Singapore for 2 weeks. A week
and a half ago, he returned to the United States to nish his nal 2 weeks at the hospital associated with his
California medical school .
The patient reports that, during his elective, several nursing staff had been sick with respiratory symptoms "at
the same time" and that the morale was poor among those who were relatively well. On further questioning,
the patient states that he had direct contact with several patients in the ICU and emergency department.
Additionally, he recalled escorting several septic patients to the radiology department for their imaging studies .
The pa ent is 5 9 in tall and weighs 155 lb (70.3 kg)
. He appears apprehensive and acutely ill, with a cough but
no sputum. His vital signs are as follows: temperature, 101.3F (38.5C) on admission to the emergency
department; blood pressure, 110 mm Hg systolic, 65 mm Hg diastolic; heart rate, 108 beats per minute; and
respiratory rate, 18 breaths per minute with no retrac ons. The pa ent's mental status is normal with a
nonfocal neurologic examination; he has no meningismus. The patient's mucous membranes are dry, and he
has a few petechiae on his upper palate and no pharyngeal exudates. The patient has diffuse bilateral rhonchi
with few bibasilar rales. Slight splinting is observed on both sides when the patient is asked to take a deep
breath. No murmur or gallop is noted. The patient has no edema or rash in the extremities. Examination
findings of the abdominal, genitourinary, and musculoskeletal areas are negative .
CBC ndings are as follows: WBC count, 2.5 X 109/L (no le shi ); lymphocyte count (LYC), 0.8 X 109/L;
hemoglobin (Hgb), 11 mg/dL; hematocrit (Hct), 33%; and platelet count, 89,000 per cubic milliliter. Chemistry
ndings are as follows: sodium, 131 mg/dL; BUN, 35 mg/dL; crea nine (Cr), 1.6 mg/dL; aspartate
aminotransferase (AST), 73 mg/dL; alanine aminotransferase (ALT), 65 mg/dL; lactate dehydrogenase (LDH), 397
mg/dL; and O2 satura on, 89%. ABG measurements are pH, 7.32; pCO2, 31 mm Hg; and pO2, 56 mm Hg.
Hint:
This is an acute, febrile, progressively worsening respiratory illness. Epidemiologic clues include the patients
recent international travel and exposure to a hospital health care worker.

3o clinical cases from eMedicine


Case 3 Answer
FEBRILE PNEUMONIA IN A MEDICAL STUDENT RETURNING F ROM SOUTHEAST ASIA
Diagnosis: Severe acute respiratory syndrome (SARS)
Discussion:
Severe acute respiratory syndrome (SARS) is probable on the basis of both the clinical and the radiographic
criteria. (Note that this working diagnosis is based on the features of the case definition used in the SARS
outbreak, ie, syndromic definition, rather than on confirmed laboratory results. Laboratory evidence is not
required to meet the criteria for defining a case of SARS).
The chest radiograph depicts bibasilar interstitial infiltrates and a hazy appearance in both lobes. No pleural
effusion or cavitation is seen. The chest CT scan depicts patchy ground-glass opacification of both lower lobes
that is most prominent in the retrocardiac region.
Although several features of this case support a range of differential diagnoses, this scenario includes some of
the distinctive characteristics of the newly described disease, SARS. The most important epidemiologic feature
is travel to one of the main countries known to have documented or suspected community transmission of
SARS, namely, Singapore (as of February 1, 2003). Other noteworthy features include the initial atypical
pneumonia with a fever, myalgias, and a dry cough, followed by increasing lower respiratory symptoms and
signs within an illness period of about 1 week. This illness also seemed to occur a er a 2- to 10-day incubation
period, which is considered the incubation period for SARS .
As a health care worker, the patient likely had direct exposure to the respiratory secretions of infected patients
in Singapore. Additionally, he had diarrhea, a less common feature. Although this finding may dissuade
clinicians form diagnosing SARS, it is noted in as many as 10% of cases. The pa ent^s leukopenia, lymphopenia,
and elevated LDH level, along with hyponatremia and an elevated ALT level, also support the working diagnosis .
Given the patient^s stable ini al course for the rst 2-3 days, followed by worsening ndings in the lower
respiratory tract, the course is consistent with the natural history of SARS observed thus far. The infectioncontrol practices that may have been in place during the student^s rotation are unclear, and appropriate
protection protocols, including use of N-95 masks, universal precau onary measures, respiratory protec on,
and negative isolation, may not have been in use. Among other staff, medical students may take greater
personal risk in caring for patients because of their idealism and altruism .
This patient smokes, and factors such as older age and comorbid conditions (eg, smoking or diabetes mellitus)
may increase a patient^s risk of severe disease progression. The radiographic features are typical for the
worsening SARS presentation. With an obvious A-a gradient, the incipient picture is that of an ominous
challenge in air-exchange resulting in shunt physiology. Laboratory tests to confirm the presence of the SARScoronavirus are available. Serologic tests must be performed on an acute and convalescent basis (>21 d a er
the onset of symptoms). At the me of this pa ent^s presenta on, which was only-14
10 days postexposure
and less than 1 week into the course of natural illness, the enzyme-linked immunosorbent assay (ELISA) results
would likely have been falsely negative and certainly not adequate to rule out coronavirus-associated SARS.

3o clinical cases from eMedicine


Case 4
LIGHTHEADEDNESS IN AN ELDERLY MAN
Background
An 80-year-old man is referred for a preoperative cardiac risk assessment prior to elective surgery for a hernia
repair. He has no personal history of coronary artery disease or myocardial infarction, and no known family
history of atherosclerotic heart disease. The patient denies having any symptoms of episodic chest pain and
pressure, palpitations, nausea, vomiting, diaphoresis, or syncope. He does, however, admit to some exertional
shortness of breath that he attributes to a long history of habitual cigarette smoking, as well as an established
diagnosis of chronic obstructive pulmonary disease (COPD). In addition to the shortness of breath, he has also
experienced brief episodes of lightheadedness from time to time. The lightheadedness occurs without warning
and without any identifiable precipitating factor, and it abates without intervention. Other than his COPD, the
patient has been remarkably healthy his whole life. His review of systems, other than as noted above, is
negative.

Figure 1

On physical examination, the patient is afebrile, with a blood pressure of 125/67 mm Hg, a pulse of 75 bpm, a
respiratory rate of 20 breaths/min, and an oxygen satura on of 91% while breathing room air. The pa ent
appears younger than his stated age. He is in no acute distress. The neck examination shows no appreciable
jugular venous distension or carotid bruits. His heart sounds are remarkable for a regular heart rhythm, with
frequent skipped beats and a slightly accentuated second heart sound. No murmurs, rubs, or gallops are
appreciated. Auscultation of his chest reveals distant breath sounds with no wheezing, crackles, or rhonchi.
There is no peripheral edema of the lower extremities. The remainder of the physical examination is
unremarkable.
A panel of preoperative blood tests and an electrocardiogram (ECG) are ordered. While the patient is waiting in
the preoperative holding area, he experiences an episode of lightheadedness. Upon noting a rapid pulse, a
technician attaches leads to obtain a cardiac rhythm strip and an ECG. His blood pressure is recorded at 80/46
mm Hg. A 12-lead ECG is obtained (see Figure 1).

3o clinical cases from eMedicine


What rhythm does the first portion of the ECG demonstrate?
Hint: Note the association between the P waves and each QRS complex.
o
o
o
o

Nonsustained ventricular tachycardia


Supraventricular tachycardia (SVT) with aberrant conduction
Atrial Flutter
Torsade de Pointes

3oo clinical cases from eMedicine


Case 4 Answer
LIGHTHEADEDNESS IN AN ELDERLY MAN

Diagnosis: Nonsustained ventricular tachycardia


Discussion:
A wide-complex
complex tachycardia is a cardiac dysrhythmia with a ventricular rate that exceeds 100 bpm in the se ng
of a QRS dura on greater than or equal to 120 milliseconds. A wide-complex tachycardia can originate from
either a ventricular focus or a supraventricular focus associated with a conduction abnor
abnormality. In this case,
based on the QRS morphology (the QRS width being greater than 140 milliseconds at the widest leads) and the
atrioventricular dissociation (see arrows), the ECG was determined to have the characteristics of ventricular
tachycardia (see subsequent discussion on how to determine the focus of a wide-complex
wide complex tachycardia).

Figure 1

Ventricular tachycardia is the most common cause of wide-complex


wide complex tachycardias, accounting for as many as
80% of cases. The frequency can be even higher in pa ents with structural or ischemic heart disease.
Ventricular tachycardia also occurs in patients with electrolyte abnormalities, such as hypokalemia and
hypomagnesemia, as well as in hypoxemic patients, individuals with acidemia, and patients with mitral va
valve
prolapse. The rhythm can occasionally occur in individuals without any identifiable risk factors. Adverse drug
reactions can also induce ventricular tachycardia by prolonging the QT interval. Drugs that are known to

3o clinical cases from eMedicine


increase the risk for ventricular tachycardia include digitalis, phenothiazines, tricyclic antidepressants, some
long-acting antihistamines, and, paradoxically, antiarrhythmics. Digitalis toxicity can also cause a rare
bidirectional ventricular tachycardia in which the QRS complexes in any given lead alternate in polarity. Lastly,
common iatrogenic causes of wide-complex tachycardia in certain settings are electronic pacemakers or
implantable cardioverter-defibrillators (ICDs) with pacemaker capability.
As mentioned above, a tachycardic rhythm with a wide QRS complex can also occur in association with a
supraventricular tachycardia with abnormal conduction, which can make differentiation of ventricular
tachycardia from supraventricular tachycardia with aberrant conduction difficult in the acute setting (especially
since both types of patients may present with similar symptoms).
The symptoms associated with a wide-complex tachycardia are typically caused by decreased cardiac output,
and they include orthostasis, hypotension, presyncope, syncope, dyspnea, and exercise limitation. Interestingly,
monomorphic ventricular tachycardia can be asymptomatic, despite the widespread belief that sustained
ventricular tachycardia always produces symptoms. Clinical symptomatology is, therefore, of limited utility in
the differentiation of ventricular tachycardia from supraventricular tachycardia.
Accurately diagnosing the underlying rhythm in a patient with a wide-complex tachycardia is critical for
determining treatment and management, especially if the patient presents emergently and is hemodynamically
unstable. This is of particular concern because medications routinely used to treat supraventricular tachycardia
can cause severe hemodynamic deterioration by inducing the relatively stable rhythm of ventricular tachycardia
to degenerate into ventricular fibrillation, with subsequent cardiac arrest. In fact, misdiagnosis of ventricular
tachycardia as supraventricular tachycardia with abnormal conduction in patients presenting with a widecomplex tachycardia is not uncommon, especially if the abnormal rhythm is hemodynamically tolerated. In
general, if the clinician is unsure, a wide-complex tachycardia should be presumed to be a ventricular
tachycardia until the presence of supraventricular tachycardia can be definitively proven. A patient with a widecomplex tachycardia in an unstable condition should receive immediate electrical cardioversion. In patients with
a stable ventricular tachycardia or with a wide-complex tachycardia of unclear origin, pharmacologic agents,
including amiodarone, procainamide, or lidocaine, may be used in accordance with established Advance
Cardiovascular Life Support (ACLS) guidelines. If a determination is made that a wide-complex tachycardia is a
supraventricular tachycardia with abnormal conduction, a trial of vagal stimulation (carotid massage) or
treatment with adenosine may be attempted.
Several studies have, by utilizing various criteria or combinations of criteria, attempted to improve the
diagnostic accuracy of differentiating ventricular tachycardia from supraventricular tachycardia in the
evaluation of wide-complex tachycardia. Although no single algorithm is 100% sensi ve and 100% specic,
several characteristics and clues can be of use. One of the most well recognized systematic algorithms consists
of 4 dieren a ng characteris cs proposed by Brugada, and it is described as follows:
If an RS complex cannot be identified in any precordial lead, ventricular tachycardia can be diagnosed with
100% specicity and 21% sensi vity.
If an RS complex is clearly distinguished in one or more precordial leads, the interval between the onset of the R
wave and the deepest part of the S wave (RS interval) is measured (if RS complexes are present in several
precordial leads, the longest RS interval is used). If the RS interval is greater than 100 milliseconds, ventricular
tachycardia can be diagnosed with 98% specicity and 66% sensi vity.
If the RS interval is less than 100 milliseconds, the presence or absence of atrioventricular (AV) dissociation
must be determined. Evidence of AV dissocia on is 100% specic and 82% sensi ve for ventricular tachycardia;
this is because atrioventricular dissociation does not occur in supraventricular tachycardia. AV dissociation is
characterized by atrial activity that is completely independent of ventricular activity. Although the presence of
AV dissociation establishes ventricular tachycardia as the etiology, its absence does not exclude the possibility
of ventricular tachycardia with retrograde conduction of ventricular impulses through the AV node producing an
atrial rhythm. This phenomenon, called retrograde ventriculoatrial conduction, is easily misinterpreted as
atrioventricular conduction because of the presence of P waves.

3o clinical cases from eMedicine


If the RS interval is less than 100 milliseconds, and if atrioventricular dissocia on cannot clearly be
demonstrated, the QRS morphology may be evaluated. Morphologic criteria suggestive of ventricular
tachycardia are extensive and complex, and they should be evaluated in conjunction with a cardiologist, if
necessary. This nal step in evalua on moves the algorithm to 96.5% specic and 98.7% sensi ve.
Other characteristic ECG findings that can be helpful for quickly differentiating ventricular tachycardia from
supraventricular tachycardia include the following:

An extreme rightward axis (-90 to -180 degrees) is o en more sugges ve of ventricular tachycardia.
A slight irregularity of the RR intervals, especially in the early stages before settling into a regular
rhythm, can be suggestive of ventricular tachycardia.
The width of the QRS complex can also be useful for distinguishing supraventricular tachycardia from
ventricular tachycardia. In general, a wide QRS complex greater than 140 milliseconds suggests
tachycardia; however, a QRS dura on of less than 140 milliseconds is not helpful for excluding
ventricular tachycardia, because ventricular tachycardia is sometimes associated with a relatively
narrow QRS complex.
If the degree of voltage change in the rst 40 milliseconds of the QRS complex is less than the degree of
voltage change in the last 40 milliseconds of the complex, this nding is sugges ve of ventricular
tachycardia.
"Fusion" occurs when a supraventricular impulse reaches the atrioventricular node simultaneously with
a ventricular impulse. The resulting QRS complex has a hybrid morphology that is between a narrow
atrial complex and a wide ventricular complex. Intermittent fusion beats during a wide-complex
tachycardia indicate atrioventricular dissociation and, therefore, also indicate ventricular tachycardia.
A "capture beat" occurs when a supraventricular rhythm briefly conducts in a normal fashion, with a
resultant normal QRS complex. The term "capture beat" implies that the normal conduction system has
momentarily replaced the control of a ventricular focus; hence, ventricular tachycardia is present.

As mentioned above, the patient in this case was diagnosed with ventricular tachycardia, and his elective
surgery for repairing the hernia was put on hold. An electrophysiology study was arranged after consultation
with a cardiologist. An arrhythmogenic focus of myocardial irritability, which was thought to be caused by scar
tissue from an unrecognized previous myocardial infarction, was identified during the study. The patient had an
automatic internal cardiac defibrillator placed and, subsequently, his hernia was successfully repaired.

3o clinical cases from eMedicine


Case 5
MAN FROM MIDDLE EAST
Background
A 19-year-old man presents to the hospital with complaints of fever, a productive cough, night sweats,
anorexia, and weight loss. He recently immigrated to Canada from Middle east. What is the diagnosis?

Figure 1

Hint
Physicians in ancient Greece called this illness phthisis.

3o clinical cases from eMedicine


Case 5 Answer
MAN FROM MIDDLE EAST
Diagnosis: Postprimary cavitary tuberculosis
Discussion:
Postprimary tuberculosis tends to localize in the apical and posterior segments of the upper lobes (85%) and in
the superior segments of the lower lobes (10%). Chest radiographs demonstrate cavita ons in 20-45% of
patients in whom the infection is reactivated.

3o clinical cases from eMedicine


Case 6
A 47-YEAR-OLD MAN WITH ACUTE EPIGASTRIC PAIN
Background
A 47-year-old man presents to the emergency department (ED) with a chief complaint of waking up with severe
abdominal pain. He states that over the past week he has had intermittent, gaslike epigastric pain and a
sensation "like I need to burp"; as of this morning, the pain has acutely worsened. It is now radiating in a
bandlike pattern throughout the patient's upper abdomen and to his back. The pain is most intense when he
lays flat on his back and seems to be slightly better when he is sitting upright. The patient has mild nausea but
has not vomited. He has not experienced any chills or fever, and he denies having any diarrhea (although the
patient did have one nonbloody bowel movement before coming to the ED). On the review of symptoms, no
chest pain, shortness of breath, or palpitations are noted. Other than a history of alcohol use (the patient drinks
approximately 6-8 beers daily), he has no chronic medical condi ons and does not take any medica ons. He
denies using over-the-counter pain medications, including nonsteroidal anti-inflammatory drugs (NSAIDs). He
does not have any medication allergies.

Figure 1

3o clinical cases from eMedicine


On physical examination, the patient is noted to be a thin, slightly emaciated man who appears uncomfortable
and in obvious distress. His vital signs include a temperature of 95.7F (35.4C), a pulse of 87 bpm, a respiratory
rate of 28 breaths/min, a blood pressure of 111/62 mm Hg, and an oxygen satura on of 98% while breathing
room air. He is diaphoretic and is writhing around in his gurney. The patient's sclerae are anicteric. The
oropharynx is clear, with slightly dry mucous membranes. The heart examination reveals a regular rhythm, with
no murmurs. The lungs are clear to auscultation in all fields, and no rales or rhonchi are found. The abdominal
examination is notable for exquisite tenderness in the epigastric and bilateral upper quadrant regions, with
focal rebound tenderness and guarding. No tenderness or palpable masses are found in the patient's lower
abdomen. The rectal examination reveals heme-negative, brown stool.

Figure 2

The pa ent is urgently placed on a cardiac monitor, and an 18-guage peripheral intravenous (IV) line is inserted
into the antecubital fossa, through which infusion of normal saline is ini ated. The pa ent is given 2 doses of IV
hydromorphone, without significant improvement in his pain or abdominal tenderness. An upright, portable
anterior/posterior chest radiograph is obtained, and it is noted to appear normal, with no air visualized under
the diaphragm. An abdominal ultrasonogram is taken that shows no evidence of gallstones or biliary wall

3o clinical cases from eMedicine


thickening; the kidneys and liver appear normal as well. All laboratory investigations, including a complete
blood count (CBC), metabolic panel, hepatic panel with lipase, and troponin, are within normal limits. An
electrocardiogram (ECG) is taken that reveals a normal sinus rhythm at a ventricular rate of 88 bpm, with
nonspecific ST flattening in the lateral leads without any change from his prior ECG. After the initial workup is
completed, an additional dose of IV pain medication is administered to the patient, which provides some relief
from his pain (although focal epigastric tenderness to palpation persists). The patient is prepped for a computed
tomography (CT) scan of the abdomen and pelvis, and oral and IV contrast images are obtained (see Figures 1
and 2).

What is the cause of the patient's acute abdominal pain?


Hint: The acute event is likely the result of an underlying pathology.
o
o
o
o

Gastroesophageal reflux disease


Acalculous cholecystitis
Acute pancreatitis
Perforated peptic ulcer

3oo clinical cases from eMedicine


Case 6 Answer
A 47-YEAR -OLD MAN WITH ACUTE EPIGASTRIC PAIN

Diagnosis: Perforated peptic ulcer


Discussion:
Transverse cuts obtained from the CT scan of the abdomen and pelvis (see Figures 1 and 2) showed free air
underneath the diaphragm consistent with a perforated viscous. The images also demonstrated fluid in the
region of the distal antrum/pylorus, with a small pocket of air in this fluid, suggesting that the stomach was the
site of the perforation. The patient's
's history of alcohol use pointed to a diagnosis of a perforated gastric ulcer.
Regarding a potential differential diagnosis in epigastric abdominal pain, several life
life-threatening etiologies that
must be recognized and treated urgently are possible. Cardiovascular
Cardiovascular etiologies, including acute coronary
syndrome and aortic dissection, must be considered, even when frank chest pain is absent. Numerous
gastrointestinal causes can present in a very similar fashion. Most commonly, a relatively benign cause, such aas
mild esophagitis and gastritis, is responsible. The pain of an uncomplicated peptic ulcer is comparable to that of
a perforated peptic ulcer, although it is typically chronic in nature. Gallbladder disease ranges from relatively
mild biliary colic to acute
te cholecystitis. Liver diseases include acute hepatitis; masses, such as abscesses or
tumors; gonococcal or chlamydial perihepatitis (Fitz-Hugh-Curtis
(Fitz
Curtis syndrome) in women; and acute cholangitis.
Acute pancreatitis may be present, with or without the presence
presence of gallbladder disease. Acute appendicitis may
first present with upper abdominal or midabdominal pain before localizing to the right lower quadrant.
Pulmonary processes, such as pneumonia, must also be considered in patients complaining of upper abdomi
abdominal
pain, even in the absence of cough or shortness of breath.
With such a broad differential diagnosis in the presentation of epigastric abdominal pain, the workup (including
laboratory investigations and radiologic imaging) has to be individualized based on age and other risk factors for
each potential disease process and on the characteristics and associated symptoms of the pain. Additionally,
repeat assessment of the symptoms and physical examination should be performed during the course of the
evaluation.
on. The administration of parenteral pain medication will often enable localization of the source of pain
and assessment of the severity of the disease; pain medication should not be withheld for fear of "masking" a
potentially serious disease process. As illustrated in this case, the patient had a somewhat vague examination
with an essentially normal initial workup, including no evidence of perforation on an upright radiograph at
presentation; this may have been dismissed as "benign" pain if it were not for
for the persistence of pain and
tenderness despite the administration of pain medication.
Uncomplicated peptic ulcer disease (PUD) is highly prevalent in the United States. When combined with
duodenal ulcers, the incidence is 1.8%, or approximately 500,000 new
new cases annually. Additionally, there are
about 4 million recurrences yearly. Approximately 90% of duodenal ulcers and 75% of gastric ulcers are
associated with Helicobacter pylori infection. Although it is still unclear, H. pylori appears to cause injury to the
stomach and duodenum through 3 poten al mechanisms, including the produc on of toxins that cause local
tissue injury, the induction of a mucosal immune response, and the increase of gastrin levels with an increase in
acid secretion. After H. pylori
ri infection, NSAIDs are the most common cause of PUD. The risk of disease and
complications (such as hemorrhage or perforation) are proportional to the daily dose taken. Advanced age and
concurrent use of anticoagulants or steroids also increase the risk for complication. Other factors that may
predispose a patient to gastric ulceration include chronic alcohol intake, smoking, and infection.

3o clinical cases from eMedicine


Perforation of a peptic or duodenal ulcer into the peritoneal cavity has the potential for significant morbidity
and mortality. The majority of cases occur in the elderly and in persons using NSAIDs. It is not uncommon that
in cases of perforated peptic ulcer, low-dose aspirin is the only NSAID taken. Remarkably, smoking appears to
be a stronger risk factor in patients under the age of 75 than NSAIDs are. Although most pa ents will give a
history of chronic epigastric pain prior to perforation, caused by the presence of the uncomplicated ulcer itself,
10-25% of pa ents will have no symptoms leading up to the perfora on. In approximately 10% of pa ents, the
perforation is accompanied by hemorrhage. Perforated peptic ulcers are most often located at the lesser
curvature of the stomach. The majority of perforated duodenal ulcers involve the anterior wall of the duodenal
bulb.
Three classic clinical stages typically occur in patients with perforated ulcers. The first stage, caused by the rapid
release of gastric juice into the peritoneal cavity, is characterized by an abrupt onset of intense abdominal pain.
The duration and intensity of this stage varies based on the size of the perforation and the extent to which
gastric juice leaks into the peritoneal cavity. During the second stage, which often does not occur, there is
spontaneous improvement in the symptoms. This likely happens as a result of fluid pouring out of the injured
intraperitoneal tissues, which causes a buffering of the acidic gastric juice. It is important to recognize that this
stage may occur; the clinician should not feel overly reassured if this occurs in a patient with an otherwise
concerning presentation. Additionally, findings of peritonitis will typically still be present despite improvement
in the patient's symptoms. The final stage of frank peritonitis is characterized by increased pain and signs of a
systemic inflammatory response. If appropriate therapy is not initiated, death soon follows.
As demonstrated in this case, the absence of free air within the abdominal cavity on a plain, upright abdominal
or chest radiograph is not noted in all cases. In fact, evidence of free air with a perforated viscous is seen in only
approximately 30% of cases. CT scanning has a much higher sensi vity for visualizing free air and may show
evidence suggesting the location of the perforation. Endoscopy must be avoided if a perforation is suspected,
since air insufflation in the stomach may open a perforation that has sealed, resulting in increased leakage of
gastric juice into the peritoneum. As an alternative to endoscopy, an upper gastrointestinal series using a watersoluble contrast agent may be performed. A perforated ulcer should initially be treated with adequate
resuscitation, which may include airway management (if indicated), the administration of adequate fluids
(especially if signs of hypovolemia or a systemic inflammatory response, such as hypotension and/or
tachycardia, are present), and pain control. Broad-spectrum antibiotics should be administered early and,
ultimately, urgent surgery is required to close the peritoneum and irrigate the peritoneal cavity.
The patient in this case became hypotensive and developed tachycardia in the ED after the completion of the
abdominal and pelvic CT scans, despite aggressive fluid resuscitation. The patient developed a fever and,
despite the administration of further rounds of parenteral pain medications, his pain worsened. No specific
etiology for the perforation could be identified on the CT scan. The consultant surgeon brought the patient to
the operating room and a large amount of bilious material was found within the abdomen during exploration, in
addition to an ulcer on the anterior surface of the antrum of the stomach, near the pylorus. The perforation was
closed with sutures, and the omentum was brought up and tacked over the perforation. The abdomen was
irrigated copiously and the skin was then closed. The patient had an uncomplicated postoperative course on IV
antibiotics, and he was discharged home after adequate recovery.

3o clinical cases from eMedicine


Case 7
DIAPHORESIS AND VOMITING IN A 62-YEAR-OLD MAN
Background
A 62-year-old man presents to the hospital with profuse diaphoresis and vomiting.

Figure 1

Hint
The pa

ent also complains of chest pressure. His blood pressure is 80 mm Hg systolic, 50 mm Hg diastolic.

3o clinical cases from eMedicine


Case 7 Answer
DIAPHORESIS AND VOMITING IN A 62-Y EAR -O LD MAN
Diagnosis: Inferior-wall acute myocardial infarction
Discussion:
Note the ST-segment elevation in leads II, III, and aVF, as well as reciprocal changes in leads I and aVL. STsegment elevations also are present in lateral leads. Findings in the right-sided leads were consistent with right
ventricular infarction; this finding was confirmed during cardiac catheterization.

3oo clinical cases from eMedicine


Case 8
LEFT LOWER QUADRANT ABDOMINAL PAIN IN A 60-YEAR-OLD MAN
Background
A 60-year-old
old man presents to the emergency department (ED) complaining of a 11-day history of crampy,
moderately intense, left-sided
sided abdominal pain. The pain is constant in nature and exacerbated by movement; it
is relieved by lying still. The patient has not experienced anorexia and has not eaten since the evening before.
He has had several loose brown stools but denies any nausea or vomiting. The stool in his bowel movements is
not blood streaked and does not appear tarry. He denies any recent travel or camping and has not eaten any
uncooked
oked or undercooked foods. He reports feeling febrile, sweaty, and generally fatigued. No urinary
symptoms, such as dysuria or increased frequency, are reported. He has not had any recent contact with sick
people. He denies having had similar episodes in the
the past. His medical and surgical histories are unremarkable,
although he did have a screening barium enema examina on 3 years ago. He is a nonsmoker and denies any
heavy or regular alcohol consumption. He does not take any prescription or over-the
over the-counter medications.

Figure 1

On physical examina on, the pa ent has an elevated temperature of 101.3F (38.5C), a blood pressure of
130/76 mm Hg, a pulse of 110 bpm, and a respiratory rate of 20 breaths/min. The pa ent is not in acute
distress, but he is mildly ill-appearing
appearing and diaphoretic.
diaphoretic. His oropharynx is clear, with slightly dry mucous
membranes. His lungs are clear to auscultation, and his heart rate is regular, without murmurs. The abdominal

3o clinical cases from eMedicine


examination reveals moderate tenderness in the left lower quadrant, with voluntary guarding. There is no
rebound tenderness. No costovertebral angle tenderness or inguinal hernias are appreciated, and his genital
exam is noted to be normal. On digital rectal examination, the patient is tender on the left side of the rectal
vault, and the stool is noted to guaiac-negative. The remainder of the physical examination is unremarkable.
Serum laboratory tes ng is remarkable only for an elevated white blood cell (WBC) count of 16.0 103/L (16.0
109/L), with a neutrophil predominance; the urinalysis is unremarkable. A standard radiograph of the
abdomen is obtained, which does not show any signicant abnormali es (see Figure 1). The pa ent then
undergoes a computed tomography (CT) scan of the abdomen and pelvis (see Figures 2 and 3).

Figure 2

3o clinical cases from eMedicine

Figure 3

What is the diagnosis?


Hint: This is the most common acute condition related to the sigmoid colon.
o
o
o
o

Acute diverticulitis
Colon cancer
Acute bacterial peritonitis
Acute appendicitis

3oo clinical cases from eMedicine


Case 8 Answer
LEFT LOWER QUADRANT ABDOMINAL PAIN IN A 60-YEAR -OLD MAN

Diagnosis: Acute diverticulitis


Discussion:
Acute diverticulitis results from inflammation of a diverticulum (small mucosal and submucosal herniations
through the circular muscle layer of the colonic wall) secondary to fecal obstruction.
obstruction. The obstruction typically
occurs at the neck of the diverticulum; solidified stool, which typically forms a fecalith, abrades the mucosa
within or at the neck of the diverticulum. In uncomplicated cases (typically characterized by a well
well-appearing
patient
atient without peritonitis and systemic signs/symptoms), the inflammatory process is confined to the colonic
wall; however, the obstruction, with subsequent high intraluminal pressure within the diverticula, can lead to a
microperforation which, in turn, allows
llows translocation of bacteria through the colonic wall, pericolic abscess
forma on, and diuse peritoni s. Only-4%
4%
2 of pa ents diagnosed with diver culi s are younger than 40 years
old; the condition is predominantly found in elderly populations.

Figure 1

3o clinical cases from eMedicine

Figure 2

Figure 3

3o clinical cases from eMedicine


The colonic diverticula themselves are most commonly found in the sigmoid and descending colon, although,
less commonly, patients develop diverticula of the right colon (particularly in patients of Asian descent). The
condition of diverticulosis is an intestinal disorder that is characterized by the presence of many diverticula and
which occurs equally in men and women, with a higher prevalence in cultures with a low-fiber diet (a low-fiber
diet is believed to decrease stool transit time, thereby causing increased intraluminal pressure and resulting in
mucosal hernia ons). Approximately one third of the popula on has diver culosis by age 50 years, and about
two thirds have it by age 85 years. Approximately 10-25% of pa ents with known diver culosis go on to develop
diverticulitis.
The classic presentation of diverticulitis consists of steady, deep abdominal pain that is often initially diffuse and
vague, but later localizes in the left lower quadrant of the abdomen. Abdominal bloating, stool changes such as
diarrhea or constipation, and flatulence frequently accompany acute diverticulitis. Fevers, fatigue, and anorexia
are also common complaints. Colonic inflammation may irritate the bladder or the ureters, leading to
complaints of urinary frequency and dysuria. A physical examination may reveal fever; localized, left lower
quadrant abdominal tenderness; mild abdominal distention; and, at times, a left lower quadrant mass. The
palpated mass is likely to be inflamed loops of bowel or, possibly, an abscess. A digital rectal examination may
demonstrate left-sided tenderness and occult blood in the stool.
The differential diagnosis of acute sigmoid diverticulitis is broad and includes inflammatory bowel disease,
irritable bowel syndrome, appendicitis, ischemic colitis, colon cancer, urolithiasis, urinary tract infection, and, in
women, a number of obstetric/gynecologic conditions (such as tubo-ovarian abscesses and ovarian cysts). The
complications of acute diverticulitis include the formation of a pericolic abscess, frank colonic perforation
leading to free intra-abdominal air, local adhesions, purulent or fecal peritonitis, sepsis, bowel obstruction, and
fistula formation between the colon and the bladder or vagina. Fistula formation is more common in the setting
of recurrent diverticulitis, with the most common type being a colovesicular fistula that is characterized by
fecaluria, pneumaturia, or typical urinary tract infection symptoms.
The initial evaluation of a patient with suspected acute diverticulitis generally includes a physical examination,
complete blood cell count, urinalysis, and, when indicated by the presence of peritonitis, plain x-rays of the
abdomen to rule out colonic perforation. Plain films are of limited value; however, they may show colonic
obstruc on, mild ileus, or bowel disten on. Leukocytosis is found in only 36% of cases of acute diver culi
The preferred imaging modality for the diagnosis of acute diverticulitis is CT scanning, as it detects both the
extent of the disease and the presence of complications. Abdominal ultrasonography can also be used, but it
lacks specificity and is operator-dependent. Barium contrast studies and colonoscopy/sigmoidoscopy should be
avoided in the setting of acute diverticulitis because of the risk of bowel perforation; however, these
examinations are often performed after resolution of the acute stage in order to evaluate for the presence of
complications, such as fistula formation or other colonic abnormalities.
The management of patients with acute diverticulitis depends upon the severity of the illness, but it is most
commonly successful with medical management alone. Well-appearing patients who are able to tolerate oral
intake and who do not have systemic symptoms, peritonitis, or complications seen on CT scans may be treated
as outpatients. In fact, reliable, nontoxic-appearing patients with a history of diverticulitis who present with
their typical symptoms may even be treated empirically as outpatients, without repeat imaging, if no significant
comorbidities (eg, an immunocompromised state, diabetes, or malignancy) exist. All patients treated at home
require close follow-up care and reexamination, and they should be given detailed return precautions for
worsening pain or systemic illness. Treatment of uncomplicated acute diverticulitis consists of bowel rest,
broad-spectrum antibiotics, and pain control. Outpatients may be instructed to begin with a clear liquid diet and
advance slowly as tolerated, whereas inpatients should be kept hydrated with intravenous fluids. Antibiotic
regimens should cover gram-negative bacteria and anaerobes. A combination of either trimethoprimsulfamethoxazole or ciprofloxacin, with either metronidazole or clindamycin, is the primary recommended
treatment regimens. Monotherapy with amoxicillin/clavulanic acid is an acceptable alternative regimen.
Patients should be admitted to the hospital if they cannot tolerate oral intake of fluids, are
immunocompromised, demonstrate signs of systemic toxicity (such as tachycardia and fever), or have
developed evidence of peritonitis or intra-abdominal complications. These patients should receive nothing by

s.

3o clinical cases from eMedicine


mouth (NPO) and should be given intravenous antibiotics. Ciprofloxacin or an aminoglycoside may be paired
with metronidazole or clindamycin as the recommended antibiotic regimen. A monotherapeutic agent, such as
piperacillin/tazobactam, ampicillin/sulbactam, or ertapenem, may also be used.
Selected abscesses detected by ultrasonography or abdominal CT scanning may be drained percutaneously,
whereas perforations, fecal peritonitis, and fistula formation all require a surgical consultation. Abscesses less
than 5 cm in diameter can be treated with an bio cs alone, although eval
uation by a surgeon should still be
sought. Recurrent diverticulitis and complicated diverticulitis are indications for partial colonic resection.
Approximately 10-25% of pa ents who are medically managed have recurrent a acks and are at an increased
risk of subsequent complica on. Interes ngly, pa ents younger than age 40 years are more likely to suer from
recurrences and are more likely to benefit from elective sigmoid resection.
In this case, the axial CT scan images of the abdomen at the level of the pelvis (see Figures 2 and 3) show acute
diver culi s of the sigmoid colon, with mul ple diver cula (arrow heads), wall thickening (arrow in Figure 3),
and inammatory stranding in the sigmoid mesentery (asterisk in Figure 2). There is no free air or abscess
forma on. The screening barium enema performed 3 years ago (Figure 4) shows mul ple diver cula in the
sigmoid and descending colon (arrowheads). As a result of systemic signs and symptoms of infection, this
patient was admitted to the hospital. He was placed on bowel rest and started on intravenous metronidazole
and ciprooxacin. Over the next 2 days, the pa ent defervesced and his leukocytosis resolved. His diet was
advanced to a full diet, and he was discharged from the hospital on a 10-day course of amoxicillin/clavulanic
acid.

3o clinical cases from eMedicine


Case 9
A 14-YEAR-OLD BOY WITH PROGRESSIVE WEAKNESS AND DYSPNEA
Background
A 14-year-old boy presents to the emergency department (ED) with a 10-day history of progressive weakness.
The patient reports experiencing rhinorrhea, cough, and malaise approximately 3 weeks before admission. He
developed lower-extremity weakness and diculty walking 8 days a er the onset of the upper respiratory tract
infection symptoms. He was evaluated at a local hospital, where he was diagnosed with dehydration, treated
with intravenous fluids, and discharged to home. Despite these measures, his lower-extremity weakness did not
improve. Over the following 7 days, he began experiencing diuse muscle pain and progressive weakness that
extended to his upper extremi es. During the 3 days before this presenta on, he developed a hoarse voice and
shortness of breath. He also notes that he is now having difficulty urinating and has decreased oral intake. He
currently denies having any fever, cough, vomiting, or diarrhea. The patient's past medical history is significant
only for attention deficit hyperactivity disorder (ADHD), for which he takes methylphenidate. He has had no
previous hospitalizations, has no known drug allergies, and has had all recommended childhood immunizations.
His family history is noncontributory.

Figure 1

3o clinical cases from eMedicine


The physical examination reveals an afebrile, ill-appearing teenager, with a heart rate of 118 bpm, a respiratory
rate of 28 breaths/min, a blood pressure of 168/122 mm Hg, and an oxygen satura on of 93% while breathing
room air. Auscultation of the lungs reveals diffuse, poor aeration. His heart sounds are normal, without any
appreciable murmur. His strength is symmetric but diminished to 2/5 in his lower extremi es and 4/5 in his
upper extremi es (5/5 being normal strength). The pa ent's sensa on is intact to light touch, but there is a loss
of vibratory sense. He has no deep tendon reflexes in his lower extremities, diminished deep tendon reflexes
(1+) in his upper extremi es, and absent plantar reexes. Cranial nerves II-XII are intact; however, he has a
weak cough and gag reflex, with impaired handling of secretions. The remainder of his examination is
unremarkable.
The patient is intubated for progressive respiratory distress and loss of airway-protective reflexes. He is fluidresuscitated with a liter of intravenous normal saline. An electrocardiogram (ECG) is obtained, which
demonstrates sinus tachycardia. The initial laboratory analysis, including a complete blood cell (CBC) count and
a basic metabolic panel, is within normal limits. A lumbar puncture is performed, with an opening pressure of
15 cm H20. The cell count and Gram stain of the cerebrospinal uid (CSF) demonstrates 2 white blood cells per
high power eld, 4 red blood cells per high power eld, and no organisms. Addi onal analysis of the CSF shows
a protein concentra on of 96 mg/dL (960 mg/L) and glucose concentra on of 72 mg/dL (3.99 mmol/L). The
patient is sent for magne c resonance imaging (MRI) of his brain and spine (see Figure 1) and is transported to
the pediatric intensive care unit (ICU) for further management.

What is the patient's condition as verified by the MRI?


Hint: Look closely at the cauda equina on the MRI images.
o
o
o
o

Spinal epidural abscess


Guillain-Barr syndrome
Transverse myelitis
Multiple sclerosis

3oo clinical cases from eMedicine


Case 9 Answer
A 14-YEAR -OLD BOY WITH P ROGRESSIVE WEAKNESS AND DYSPNEA

Diagnosis: Guillain-Barr
Barr syndrome
Discussion:
The lumbrosacral MRIs (see Figures 1 and 2) demonstrate nerve root enhancement of the cauda equina on axial
post-contrast T1-weighted
weighted sequences. The localization of progressive weakness includes spinal cord lesions
(such as transverse myelitis or anterior spinal artery syndrome), peripheral
peripheral neuropathies (such as those caused
by heavy metals), neuromuscular junction diseases (such as that caused by organophosphate pesticides),
myasthenia gravis, botulism, and myopathies (such as dermatomyositis). The presence of progressive ascending
weakness, areflexia, autonomic dysfunction, elevated CSF protein without pleocytosis, and enhancement of the
cauda equina nerve roots on lumbrosacral MRIs make the diagnosis of Guillain-Barr
Guillain Barr syndrome most probable
in this patient.

Figure 1

3o clinical cases from eMedicine

Figure 2

Guillain-Barr syndrome is an acute, idiopathic, monophasic, acquired inflammatory demyelinating


polyradiculoneuropathy (AIDP) that affects both children and adults. It is a heterogeneous syndrome, with
several variant forms. AIDP is the prototype of Guillain-Barr syndrome, and it is the most common form in
North America, Europe, and most of the developed world (where it accounts for about 85-90% of cases).
Guillain-Barr syndrome can occur at any age, but there appears to be a bimodal distribution, with peaks in
young adulthood (15-35 y) and in the elderly (50-75 y). The cause of Guillain-Barr syndrome is unknown, but
the disorder is thought to result from a postinfectious immune-mediated process called molecular mimicry that
predominantly damages the myelin sheath of peripheral nerves. Approximately two thirds of patients report a
history of an antecedent respiratory tract or gastrointes nal infec on-42 weeks before the onset of neurologic
symptoms. A variety of infectious agents have been associated with Guillain-Barr syndrome, although
Campylobacter is the most frequent. Other organisms that commonly precede Guillain-Barr syndrome include
cytomegalovirus, Epstein-Barr virus, Haemophilus influenzae, Mycoplasma pneumoniae, the enterovirus family,
hepatitis A and B, herpes simplex virus, and Chlamydophila (formerly Chlamydia) pneumoniae.
The typical presentation of Guillain-Barr syndrome is fine paresthesias in the toes and fingertips, followed by
symmetric lower-extremity weakness that may ascend, over hours to days, to involve the arms and the muscles
of respiration. Pain, predominately back, lower-limb and abdominal pain, is often a prominent feature of the
syndrome. The physical examination reveals symmetric weakness, with diminished or absent reflexes and
variable loss of sensation in a stocking-glove distribu on. Signs of autonomic dysfunc on are present in 50% of
patients, and they include cardiac dysrhythmias, orthostatic hypotension, transient or persistent hypertension,
ileus, constipation, and bladder dysfunction. Deviation from the classic presentation of ascending progression of

3o clinical cases from eMedicine


weakness is not uncommon. In what is known as the Miller-Fisher variant, cranial nerves are aected in 30-40%
of patients at any time in the course of the syndrome. This form of the disease is also characterized by areflexia,
ataxia and ophthalmoplegia. The facial nerves are most commonly involved, resulting in bilateral facial
weakness.
Although the associated autonomic dysfunction may produce life-threatening complications, mortality from
Guillain-Barr syndrome is largely secondary to respiratory failure associated with respiratory muscle weakness.
Approximately 20% of children with Guillain-Barr syndrome require mechanical ventilation for respiratory
failure. The need for intubation should be anticipated early so that it can be done nonemergently in a controlled
environment. Progression to respiratory failure has been predicted in patients with rapid disease progression,
bulbar dysfunction, bilateral facial weakness, or dysautonomia. Emergent intubation should be performed in
any patient with loss of the gag reflex, declining respiratory function, or pharyngeal dysfunction. Care should be
taken during intubation, as autonomic dysfunction may complicate the use of vasoactive and sedative drugs.
A er the rst week of symptoms, analysis of the CSF typically reveals normal opening pressures, fewer than 10
white blood cells per high power field (typically mononuclear), and an elevated protein concentration (greater
than 45 mg/dL). This nding, also known as albuminocytologic dissocia on, may be delayed. As a result, a
repeat lumbar puncture may be required as the protein values may not rise for 1-2 weeks, and maximum
protein values may not be seen for 4-5 weeks. In addi on, gadolinium-enhanced lumbosacral MRI may
demonstrate enhancement of the cauda equina nerve roots. This imaging modality has been described to be
83% sensi ve for acute Guillain
-Barr syndrome, and abnormalities are present in 95% of typical cases.
Electrophysiologic studies are the most specific and sensitive tests for confirming the diagnosis. Most patients
demonstrate slowing of nerve conduc on 2-3 weeks a er the onset of symptoms. There are a variety of
abnormalities seen in Guillain-Barr syndrome that indicate evolving multifocal axonal demyelination in
peripheral nerves, spinal roots and/or cranial nerves. Abnormalities seen on electromyography include partial
motor conduction block, slowed nerve conduction velocities, abnormal temporal dispersion, and prolonged
distal latencies. The earliest finding, which may be present within days of symptom onset, is prolongation or
absence of the F responses, which indicates demyelination involving the proximal nerve roots.
Any patient presenting with a clinical picture consistent with Guillain-Barr syndrome requires immediate
hospitalization. The indications for admission to the ICU include, but are not limited to, respiratory insufficiency
or failure, loss of airway-protective reflexes, and severe autonomic instability.
The main modalities of therapy for Guillain-Barr syndrome include plasma exchange and intravenously
administered immunoglobulin (IVIG). Corticosteroids have not been shown to be beneficial. The American
Academy of Neurology issued a practice parameter regarding immunotherapy for Guillain-Barr syndrome that
concluded IVIG and plasma exchange are options for children with severe disease and should be reserved for
those with the following findings:
Rapidly progressing weakness
Worsening respiratory status or need for mechanical ventilation
Significant bulbar weakness
Inability to walk unaided
Several trials have demonstrated that IVIG is at least as effective as plasma exchange in the treatment of
Guillain-Barr syndrome and is associated with a lower rate of complica ons. IVIG administered at 0.4 g/kg/day
for 5 days has been shown to hasten recovery and lower the relapse rate. Doses of 1 g/kg/day over 2 days have
also been demonstrated to hasten recovery time, but early relapses are more prevalent. The combination of
plasma exchange and IVIG does not improve outcomes or shorten the duration of illness.
Complications associated with Guillain-Barr syndrome include arrhythmia, sepsis, pneumonia, ileus, deep
venous thrombosis and pulmonary embolism. The risk of sepsis and infection may be decreased by aggressive

3o clinical cases from eMedicine


physiotherapy and mechanical ventilation with positive end expiratory pressure (PEEP). Administration of
anticoagulant therapy and intermittent pneumatic compression devices may lower the risk of deep venous
thrombosis and pulmonary embolism. Cardiac telemetry is useful to monitor for arrhythmias, which are a
common cause of morbidity and mortality in Guillain-Barr syndrome. In addition, physical and occupational
therapy should be initiated early and may be beneficial in helping patients to regain their baseline functional
status.
More than 90% of pa ents reach the nadir of their func on within 4 weeks of the onset of symptoms, with
return of normal function occurring slowly over the course of weeks to months. The majority of patients with
Guillain-Barr syndrome achieve a full and func onal recovery within 6-12 months. The clinical course of
Guillain-Barr syndrome in children is shorter than it is in adults, and recovery is more complete.
Upon transferring this patient to the ICU, a dialysis catheter was placed and plasmapheresis was initiated. His
hypertension was controlled with a nicardipine drip. Prophylaxis for deep venous thrombosis was started. On
hospital day 3, the pa ent had improved strength in his lower extremi es and pressure support trials on the
ven lator were ini ated. On hospital day 6, he was extubated successfully to room air a er receiving a total of
5 sessions of plasmapheresis. He was transferred to a pediatric ward 1 week a er admission with intensive
physical and occupational therapies. At the time of the transfer, the patient's bulbar symptoms were resolved,
and his strength was 3/5 in his lower extremi es and 4/5 in his upper extremi es.

3o clinical cases from eMedicine


Case 10
AN ELDERLY MAN WITH DYSPNEA
Background
A 65-year-old man presents to the emergency department (ED) complaining of difficulty breathing.
He describes worsening dyspnea on exertion that is associated with chest tightness, wheezing, and coughing.
The patient's dyspnea has worsened to the point that he can hardly walk from his couch to the bathroom
without becoming extremely short of breath. He recently recovered from a cold, with several days of nasal
congestion, clear rhinorrhea, and a nonproductive cough. He reports having been healthy his whole life and has
not been to see a physician in at least 2 decades; however, he does admit that he has gradually curtailed his
physical activities, such as gardening, shoveling snow, and walking in the mall, because he has been increasingly
"ge ng winded." He smokes 2 packs of cigare es daily, a habit he has been trying to break for at least 30 years
.
On physical examination, the patient is alert but appears to be in mild respiratory distress, with moderate
retractions and pursed-lipped breathing. He is afebrile. His blood pressure is 140/85 mm Hg, and his pulse rate
is 103 bpm and mostly regular. His respiratory rate is 28 breaths/min, and a pulse oximetry reading shows 85%
while the patient is breathing room air. His breath sounds are diminished throughout, with a markedly
prolonged expiratory phase and faint expiratory wheezes in the upper lung fields. The cardiac examination
reveals distant heart sounds with a somewhat prominent P2. He has no murmur, gallop, or pericardial rub. His
skin is cool and dry. He has trace edema at his ankles but no cyanosis or clubbing. An electrocardiogram (ECG) is
performed (see Figure 1).

Figure 1

What is the diagnosis?


Hint: The ECG results suggest a chronic condition.
o Pulmonary embolus
o Chronic obstructive pulmonary disease
o Acute bacterial pneumonia
o Stable angina

3oo clinical cases from eMedicine


Case 10 Answer
AN ELDERLY MAN WITH DYSPNEA

Diagnosis: Chronic obstructive pulmonary disease


Discussion:
The ECG demonstrates a constellation of findings that suggest COPD, including sinus tachycardia, a rightward
axis, P pulmonale (a P wave amplitude > 2.5mV in the inferior leads), a low QRS voltage, and a right bundle
branch block (RBBB).
In addition, the slight ST segment depressions
depressions in the inferior leads are suggestive of prominent atrial
repolarization abnormalities and are seen in COPD.
COPD

Figure 1

Chronic obstructive pulmonary disease (COPD) is currently the fourth leading cause of death in the United
States. COPD is defined as a disease state characterized by airflow obstruction caused by chronic bronchitis or
emphysema. The airflow obstruction generally is progressive, and it may be accompanied by partially reversible
airway hyperreactivity. The condition was rst described in Western Europe in the early 19th century by
Badham (1808) and Laennec (1827), who made the classic descrip on of chronic bronchi s and emphysema. A
Bri sh medical textbook of the 1860s described the familiar clinical picture of chronic bronchitis as an advanced
disease, with repeated bronchial infections, that ended in right heart failure. The modern definition of chronic
bronchitis and emphysema which incorporated the concept of airflow obstruction was proposed by participants
of the Ciba symposium of 1958.

3o clinical cases from eMedicine


Chronic bronchi s is dened, in clinical terms, as the presence of a chronic produc ve cough for 2 consecu ve
years, las ng for at least 3 months during each year, at the exclusion of other e ologies. Emphysema is dened,
in terms of anatomic pathology, as an abnormal, permanent enlargement of the air spaces distal to the terminal
bronchioles, accompanied by destruction of their walls and without obvious fibrosis. The characteristic
histopathology of chronic bronchitis is mucous gland hyperplasia with bronchial wall thickening, focal squamous
metaplasia, ciliary abnormalities, and variable amounts of airway smooth muscle hyperplasia and inflammation.
These changes lead to luminal occlusion causing airflow limitation by allowing airway walls to deform and
narrow the airway lumen. Emphysema has 3 morphologic pa erns: centriacinar emphysema (focal destruc on
limited to the respiratory bronchioles and the central portions of acinus; associated with cigarette smoking),
panacinar emphysema (involves the entire alveolus distal to the terminal bronchiole; generally develops in
pa ents with homozygous alpha1-antitrypsin [AAT] deficiency), and distal acinar emphysema or paraseptal
emphysema (the least common form, involving distal airway structures, alveolar ducts, and sacs; it is localized
to fibrous septa or to the pleura and leads to formation of bullae leading to pneumothorax).
In the United States, approximately 14.2 million people have COPD; approximately 12.5 million of these cases
stem from chronic bronchi s and the remaining 1.7 million stem from emphysema. Since 1982, the number of
pa ents diagnosed with COPD increased by 41.5%. The prevalence of chronic airow obstruc on in the United
States is es mated at 8-17% for men and 10-19% for women. Interna onally, the rates likely are higher because
more than 1.2 billion humans are exposed to the ravages of smoking. Absolute mortality rates for pa ents in
the United States aged 55-84 years (1985) were 200 per 100,000 males and 80 per 100,000 females.
Internationally, a marked variation in overall mortality rates from COPD exists. The extremes are the more than
400 deaths per 100,000 males aged 65-74 years in Romania and the fewer than 100 deaths per 100,000 in
Japan. Patients with COPD are susceptible to many conditions that can rapidly lead to an acute deterioration
superimposed on chronic disease. Quick and accurate recognition of these patients, along with aggressive and
prompt intervention, may be the only action that prevents frank respiratory failure.
The disease is not generally diagnosed on the basis of ECG findings; however, the signs and symptoms of cardiac
and pulmonary disease may overlap substantially, and ancillary testing can be useful in establishing the
diagnosis. In fact, it may not be unusual for ECG to be the first diagnostic test performed in patients with longstanding COPD, if patients present with shortness of breath as part of a general workup for a possible cardiac
etiology for the symptoms. Knowledge of the usual ECG manifestations of COPD enables the clinician to
recognize uncharacteristic abnormalities, which often represent the effects of superimposed illnesses or drug
toxicity.
A tachycardic rhythm is common in individuals who are experiencing exacerbations of their COPD as a
compensatory mechanism for hypoxia or poor right ventricular function (in the setting of cor pulmonale). Sinus
tachycardia is the most common form reported in the literature, but other supraventricular arrhythmias, such
as atrial tachycardia (unifocal or multifocal), atrial fibrillation, and atrial flutter, can also be present.
P pulmonale (ie, a P wave amplitude >2.5 mm) is frequently reported but is a rela vely insensi ve predictor of
right atrial enlargement. In patients with COPD, the amplitude of the P wave is in fact dynamic, and it tends to
be more prominent during acute exacerbation than at other times.
A vertical or rightward axis is another manifestation of pulmonary hypertension. Similarly, complete or
incomplete RBBB, right ventricular hypertrophy, or both commonly occur in patients with cor pulmonale.
Low voltage, particularly in the limb leads, is another ECG characteristic of patients with COPD. This finding is
classically attributed to increased impedance through a hyperinflated chest; however, low voltage is not directly
correlated with hyperinflation, and it is neither sensitive nor specific for COPD.
The mainstays of therapy for acute exacerbations of COPD, as in this case, are oxygen, bronchodilators, and
steroids. Adequate oxygen should be given to relieve the hypoxia. Supplemental oxygen should maintain a nearnormal satura on to above 90%. Bronchodilator therapy with both beta-agonist and anticholinergic nebulizer
therapy should be administered promptly. The need for intubation should be established and performed if
indicated (e.g., hypoxia not relieved with supplemental oxygen, severe respiratory distress, obtundation). If

3o clinical cases from eMedicine


necessary and available, continuous positive airway pressure (CPAP) may be used. Other therapies that are
indicated in specific situations may include antibiotics, magnesium, and Heliox (a mixture of helium and oxygen
that leads to increased oxygen delivery as a result of improved laminar flow).
The patient in this case was given a combination beta-agonist and anticholinergic nebulizer therapy for his
respiratory distress and hypoxia. A chest radiograph was performed, which revealed a moderately sized basilar
infiltrate. Antibiotics appropriate for community-acquired pneumonia coverage and intravenous steroids were
administered, and the patient was admitted to the hospital for continued inpatient therapy and monitoring. On
the morning of hospital day 3, the pa ent was noted to be breathing easier and was discharged to home with
prescriptions for maintenance inhaled steroid therapy, and for beta-agonist and anticholinergic metered-dose
inhalers (MDIs). Appropriate follow-up was arranged for continued outpatient management.

3o clinical cases from eMedicine


Case 11
WEAK ALL OVER
Background
A 55-year-old woman with type 2 diabetes mellitus states that she has been feeling "weak all over for a few
days." She has generalized edema, including edema of the hands, legs, and eyelids.

Figure 1

Hint
You could hit a golf ball off of those T waves.

3o clinical cases from eMedicine


Case 11 Answer
WEAK ALL OVER
Diagnosis: Hyperkalemia
Discussion:
In the setting of hyperkalemia, the T waves may start to become increasingly tall and peaked at a serum
potassium level of 5.5 mmol/L. The QRS complex may begin to widen at a serum potassium level of 6.5 mmol/L.
The P wave may begin to flatten at a serum potassium level of 7 mmol/L. Note the PR prolonga on. PR
prolongation caused by hyperkalemia is usually primarily due to an increase in the P wave duration.
This patient has a first-degree atrioventricular (AV) block. At a serum potassium level of 8 mmol/L, the P wave
may become invisible. This pa ent had a serum potassium level of 7.1 mmol/L and a pH of 7 in the se ng of
renal failure. Her generalized edema was the result of a 25-lb weight gain due to the renal failure.

3o clinical cases from eMedicine


Case 12
THE UNRESTRAINED DRIVER
Background
A 26-year-old man with an unknown past medical history arrives to the emergency department (ED) by
ambulance. He had been driving his car while unrestrained and was involved in a high-speed motor vehicle
collision. There was airbag deployment and significant front-end damage to the vehicle, with intrusion into the
passenger compartment of the car. The patient was extricated from the vehicle and placed on a backboard, and
a cervical collar was placed by EMS. A non-rebreather facemask and 1 peripheral intravenous (IV) line were
placed in the field.
On arrival to the hospital, the patient is ill-appearing and comba ve. His ini al vital signs are a heart rate of 117
bpm, a blood pressure of 85/50 mm Hg, a respiratory rate of 32 breaths/min, and an oxygen satura on of 91%
on the non-rebreather mask. On primary survey, his oropharynx is clear, his airway is patent, and his trachea
appears to be shifted to the right of midline. On auscultation, the patient's breath sounds are decreased over
the left chest. Percussion of the left chest demonstrates hyperresonance. His carotid pulse is weakly palpable,
and his jugular venous pulse is elevated. The pa ent receives a Glasgow Coma Scale score of 12. The pa ent's
clothing is removed, revealing no obvious deformities or areas of bleeding. The patient's abdomen is soft,
without any tenderness to palpation. His pelvis is stable. Standard trauma x-rays, including an anteroposterior
(AP) chest and pelvis scan, are performed after the primary survey. A complete secondary survey is postponed
because of the patient's poor clinical condition.

Figure 1

3o clinical cases from eMedicine

A second large-bore peripheral intravenous line is placed, and the pa ent begins to receive a bolus of 1000 cc of
normal saline under pressure. A decision to perform an emergent procedure is made. Immediately after the
procedure is performed, the patient is noted to have a dramatic clinical improvement. Subsequent to the
procedure, the pa ent has a pulse of 105 bpm, a blood pressure of 95/60 mm Hg, a respiratory rate of 22
breaths/min, and an oxygen satura on of 98% on the non-rebreather mask. The secondary survey is completed,
revealing no major injuries. Addi onally, the chest radiograph (see Figure 1) conrms the suspected clinical
diagnosis that prompted the emergent procedure.

What is the underlying pathophysiology, and what procedure was performed?


Hint: The cause of this patient's hypotension and hypoxia is a clinical diagnosis, and although a portable chest
radiograph was performed in this case, this condition should not typically require imaging.
o
o
o
o

Upper airway obstruction; cricothyrotomy


Tension pneumothorax; needle thoracostomy
Hypovolumic shock; central line placement
Pericardial tamponade; pericardiocentesis

3oo clinical cases from eMedicine


Case 12 Answer
THE UNRESTRAINED DRIVER

Diagnosis: Tension pneumothorax; needle thoracostomy


Discussion:
Pneumothorax occurs when air enters the potential space between the visceral and parietal pleura, leading to
lung collapse on the affected side. Pneumothoraces may occur spontaneously, especially in the setting of lung
disease, or they may result from accidental or iatrogenic trauma. A tension pneumothorax is a life
life-threatening
condition that occurs when the air in the pleural space is under pressure, displacing mediastina
mediastinal structures and
compromising cardiopulmonary function. Tension pneumothoraces result from injuries to the lung parenchyma
or bronchial tree that can act as one-way
one way valves so that air enters the pleural space but cannot escape. The
trapped air in a tension pneumothorax causes increased intrathoracic pressure, pushing mediastinal structures
contralaterally and reducing venous return and cardiac output. These patients are hypoxic and become difficult
to ventilate, with potentially rapid progress to cardiorespiratory
cardiorespiratory collapse and death. Hemothorax is defined by
blood in the pleural space, and it occurs when the lung parenchyma and the intercostal or mammary vessels are
injured. Massive hemothoraces arise with hilar injuries, aortic ruptures, or myocardial ruptu
ruptures. A tension
hemopneumothorax develops when there is both blood and air under tension in the pleural space
space.
A pneumothorax in any patient who has sustained thoracic trauma should arouse suspicion. The patient may
complain of an acute onset of sharp pleuritic
pleuritic chest pain, with radiation to the ipsilateral shoulder and
associated dyspnea and anxiety. Typical physical findings in pneumothorax include unilaterally decreased
breath sounds, hyperresonance to percussion over the affected lung, and asymmetric ches
chest rise. In tension
pneumothorax, the patient displays respiratory distress, tachypnea, and tachycardia, and the patient may also
experience cyanosis, jugular venous distention, tracheal deviation away from the affected lung, and a pulsus
paradoxus.
The epidemiology
demiology of traumatic pneumothoraces has not been well characterized. In the United States, trauma is
the leading cause of death in persons younger than 45 years, and it accounts for approximately 150,000 deaths
annually. The overall mortality for thoracic
thoracic trauma is 10%, and chest injuries cause approximately 1 in 4 trauma
deaths in North America. Pneumothorax is a serious complication of thoracic trauma, and it has been described
in 1 in 5 pa ents that survive major trauma. Interes ngly, in one study,2%
1 of pa ents with asymptoma c
chest stab wounds had a delayed pneumothorax or hemothorax.
While pneumothoraces in stable patients can be confirmed radiographically, a tension pneumothorax causing
hemodynamic compromise should be diagnosed clinically, and treatment should never be delayed in favor of
diagnostic imaging. A chest x-ray
ray may show a linear shadow of visceral pleura, without lateral lung markings. An
upright chest x-ray
ray is more sensitive than a supine radiograph, as air tends to accumulate at the lung apex. In
recumbent patients, air often accumulates in the anterior portion of the inf
inferior chest and manifests
radiographically as a "deep sulcus." If a pneumothorax without tension physiology is suspected but not seen on
the initial upright chest x-ray,
ray, a repeat film during exhalation may reveal it. Increasingly, ultrasound is being
used as a rapid bedside modality for diagnosing pneumothoraces; some studies have shown that it is more
sensitive than radiography for detecting traumatic pneumothoraces. Computed tomography (CT) is more
sensitive and specific than chest x-rays
x
or ultrasonography
hy for the evaluation of small pneumothoraces and
hemothoraces. Occult pneumothoraces may be present in 2-55%
2 55% of trauma pa ents, although the clinical

3o clinical cases from eMedicine


significance of occult pneumothoraces in patients who are not mechanically ventilated under positive pressure
is unclear. Making the diagnosis of hemothorax may be more challenging. A minimum of 200-300 mL of blood is
needed in the pleural space for blunting of the costophrenic angle to be visible on an upright chest x-ray. Blood
is more difficult to appreciate on a supine x-ray because it will typically layer posteriorly, and ever larger
volumes (up to 1000 mL) of blood may produce only a mild diuse radiodensity. Lateral chest lms may help
differentiate hemothoraces from pulmonary contusions, and ultrasonography may also be useful for detecting
fluid above the diaphragm. As with pneumothoraces, CT scanning is the most sensitive modality for diagnosing
hemothoraces, although patients with massive hemothoraces may be too unstable for the scan.
The treatment of traumatic pneumothoraces and hemothoraces depends upon the volume of blood or air that
has accumulated and on the condition of the patient. Hemodynamically stable patients who are not intubated
and have a rela vely small pneumothorax (ie, less than 1cm wide) can be placed under observation. A repeat
lm should be obtained a er 4-6 hours; if the pneumothorax is unchanged in size, the pa ent can con nue to
be observed without the need for decompression or tube thoracostomy. These patients should always be
placed on 100% oxygen to increase the rate of reabsorp on of the air in the pleural space. In unstable pa ents
who, on clinical grounds, are suspected of having a pneumothorax, a needle thoracostomy may be performed
to quickly decompress the pleural space. A 14-gauge Angiocath (18-gauge or 20-gauge in an infant) should be
placed immediately superior to the rib in the second intercostal space, midclavicular line on the affected side.
Once in place, the needle is removed and the Angiocath is secured. A rush of air may be appreciated as the
Angiocath enters the pleural space. Pneumothoraces should preferentially be decompressed either by needle
decompression or placement of a tube thoracostomy before the patient is intubated, as positive pressure
ventilation will exacerbate a pneumothorax; however, definitive management of the airway should never be
delayed when indicated. Needle thoracostomy generally necessitates the subsequent placement of a chest
tube; however, stable patients who do not require a chest tube may be observed. In simple spontaneous
pneumothoraces, a 20F or 22F chest tube may be used; however, larger-caliber chest tubes (28F to 40F) should
be used in most traumatic pneumothoraces and hemothoraces to ensure adequate drainage of any fluid. Chest
tubes are placed in the fourth or fifth intercostal space in the anterior axillary or midaxillary line, and they
should be directed posteriorly and toward the apex of the lung. After the tube is secured, it should be
connected to a water seal and vacuum device, and placement should be confirmed by chest x-rays. In the case
of a hemothorax, immediate drainage of more than 1500-2000 mL (or 20 mL/kg) of blood, or ongoing
hemorrhage exceeding 600-1200 mL/6 hours (or >3 mL/kg/hr) a er the ini aldrainage, constitute the
definition of a massive hemothorax and generally are indications for a thoracotomy. Occasionally, placement of
an additional chest tube may be necessary to assist in draining of the hemothorax. Additionally, the possibility
of a bronchial injury should be considered if a continuing air leak is observed after several chest tubes and an
unexpanded lung. In hemothorax, chest tubes should be directed posteriorly and inferiorly to arrive posterior to
the diaphragm (as opposed to the placement for a simple pneumothorax).
In this case, the junior emergency medicine resident placed a 14-gauge Angiocath in the second intercostal
space, midclavicular line of the left chest. A rush of air was appreciated, and the patient's blood pressure (as
previously noted in the case presenta on) improved to 95/60 mm Hg. The resident then prepared the le chest
and placed a 38F chest tube in the h intercostal space, midaxillary line. There was immediate drainage of
1600 mL of bloody uid through the chest tube. Uncrossmatched blood was administered, and the surgical
team was consulted for the massive hemothorax. The patient was intubated and transported to the operating
room (OR). In the OR, the surgery team performed a thoracotomy, repaired the injured lung parenchyma, and
ligated several small arteries that were actively bleeding. The patient was transported to the surgical intensive
care unit (ICU) and extubated the following day. The chest tube was removed 48 hours later, and the pa ent
was discharged on hospital day 4 in stable condi on
.

3o clinical cases from eMedicine


Case 13
CHRONIC COUGH
Background
A 47-year-old man presents with a chronic cough and chest discomfort that have lasted a few months. He does
not have a fever, change of voice, or sputum production. He has no history of asthma and does not smoke. The
clinical examination revealed generalized bronchial breathing. The findings were otherwise essentially normal.
The medical history includes carcinoma of maxilla, which was treated with surgery and radiation therapy a few
years ago.
A chest radiograph was obtained.

Figure 1

What is the diagnosis?


Hint
The medical history of this patient is significant.

3o clinical cases from eMedicine


Case 13 Answer
CHRONIC COUGH
Diagnosis: Metastasis to the lungs (cannonball lesions).
Discussion:
The chest radiograph depicts multiple opacities in both lung fields consistent with pulmonary metastases. These
metastases are commonly known as cannonball lesions because of their appearance. Treatment includes
referral to an oncologist and usually chemotherapy, depending on the primary malignancy.

3o clinical cases from eMedicine


Case 14
FEVER, COUGH, AND NIGHT SWEATS
Background
A 45-year-old man complains of a fever, a cough productive of rust-colored sputum, and night sweats that have
lasted 2 weeks. He is homeless and has alcoholism. What is the diagnosis?

Figure 1

Hint
What is that straight line in the right lower lung field?

3o clinical cases from eMedicine


Case 14 Answer
FEVER, COUGH, AND NIGHT SWEATS
Diagnosis: Cavitary lesion, probable lung abscess
Discussion:
A circular lesion can be seen in the right lower lung field with the fluid level. This finding suggests the presence
of a cavity filled with fluid. Lung abscesses are found in dependent regions of the lung such as the right lower
lobe. The most common etiology is aspiration. If left untreated, this infection can extend into the plural space
(ie, empyema). Approximately 90% of lung abscesses can be treated with an bio cs and do not require surgical
evacuation. Antibiotics should include coverage for Bacteroides, Peptostreptococcus, and Fusobacterium
species. First-line agents include clindamycin, cefoxitin, and piperacillin/tazobactam (Zosyn). If the symptoms
fail to resolve, the patient should undergo an evaluation for a possible neoplasm.

3o clinical cases from eMedicine


Case 15
DOCTOR! THE PATIENT'S BLOOD PRESSURE IS FALLING!
Background
A 67-year-old woman with a history of end-stage chronic obstructive pulmonary disease (COPD) is admitted to
the hospital because of severe respiratory distress. She requires intubation in the emergency department
because of respiratory acidosis. The patient's mechanical ventilator setting is in assist control (AC) mode, with
the respiratory rate at 16 breaths per minute, the dal volume at 500 mL, and the frac on of inspired oxygen
(FIO2) at 80%. Her vital signs are temperature, 100.4F (38C); heart rate, 104 beats per minute (bpm); pulse
oximetry reading, 90%; and blood pressure, 130 mm Hg systolic, 80 mm Hg diastolic.

Figure 1

Two hours later, the nurse pages you because the pa ent's blood pressure had decreased to 75 mm Hg systolic,
50 mm Hg diastolic. Physical examina on reveals diuse and bilateral wheezing with dis nc ve heart sounds. A
frontal chest radiograph (A) is obtained.

What should be performed immediately to reverse the hypotension?


A.
B.
C.
D.

Starting a dopamine infusion


Placing a chest tube
Decreasing the ventilator rate
Increasing the tidal volume

Hint
Chest radiograph A reveals hyperinflated lung fields with the endotracheal (ET) tube properly positioned.

3o clinical cases from eMedicine


Case 15 Answer
DOCTOR! THE PATIENT'S BLOOD PRESSURE IS FALLING!
Diagnosis: Hypotension secondary to severe respiratory distress
Discussion:
The correct answer is C, decreasing the ventilator rate. The cause of this patient's hypotension is the auto
positive end-expiratory pressure (auto-PEEP) secondary to the hyperinflated lungs induced by a rapid
ventilatory rate. Venous return is decreased because of elevated intrathoracic pressures, which cause a
decrease in cardiac output and blood pressure. Auto-PEEP, intrinsic positive end-expiratory pressure (PEEP),
occult PEEP, and positive end-expiratory alveolar pressure are terms that have been used to describe the failure
of alveolar pressure to return to zero (atmospheric pressure) at the end of exhalation.
Auto-PEEP frequently occurs in mechanically ventilated patients, especially those with asthma or COPD. It is the
consequence of the ventilator delivering a positive-pressure breath before the patient has had time to
completely exhale the previous breath.
When a patient is intubated, a number of factors can contribute to the development of auto-PEEP: the
ventilator rate, the end-inspiratory pause, and the inflation volume. Careful monitoring and repeated evaluation
of high-risk patients are helpful to identify the problem.
The expiratory flow waveform should be monitored, and if the flow does not return to baseline before the
delivery of the next breath, auto-PEEP is present. If auto-PEEP is not recognized, inappropriate clinical decisions
(in terms of diagnostics and interventions) may be made.
A 2-minute trial of cessation of mechanical ventilation or decreasing the ventilator rate should be performed.
The diagnosis is confirmed if the trial results in rapid improvement in hemodynamics.
In general, auto-PEEP can be diminished by different means: decreasing the ventilatory frequency, decreasing
the tidal volume, increasing the expiratory duration, sedating the patient, performing frequent suctioning, and
using bronchodilators.
Starting a dopamine infusion in this patient would not treat the cause of the hypotension, and placing a chest
tube is indicated only in the case of a pneumothorax, which is one of the complications of auto-PEEP.
During auto-PEEP, a portion of the tidal volume is trapped in the alveoli. Therefore, the answer is to decrease
rather than increase the ventilator tidal volume.
In this case, the flow and pressure waveforms revealed an auto-PEEP of 12 cm H2O. The respiratory rate was
decreased to 10 breaths per minute, and another chest radiograph (B) was obtained shortly afterward. The
pa ent^s blood pressure increased and stabilized at 130 mm Hg systolic/ 70 mm Hg diastolic.

3o clinical cases from eMedicine


Case 16
ABDOMINAL PAIN, VOMITING, AND DIARRHEA
Background
A 76-year-old man presents with nausea, vomiting, diarrhea, and worsening abdominal pain. The pain is poorly
localized and colicky and has lasted for 2 days.

Figure 1

Hint
Signicant features of the history of this pa
coronary artery bypass surgery.

ent include an appendectomy 12 years ago,hypertension, and

3o clinical cases from eMedicine


Case 16 Answer
ABDOMINAL PAIN, VOMITING, AND DIARRHEA
Diagnosis: Early small-bowel obstruction (SBO)
Discussion:
The radiograph was obtained with the patient in an upright position; therefore, it reveals multiple loops of
small bowel filled with fluid. Straight lines of fluid can be seen in the left upper quadrant, and a single line of
fluid can be seen in the right lower quadrant. No air is depicted in the colon or rectal region. The small bowel
with the fluid is not yet dilated; this finding indicates that the obstruction is still early in its course. The most
common causes of SBO are postopera ve adhesions (60%). The most common procedures that result in
adhesions are appendectomy, colorectal surgery, gynecologic procedures, and upper gastrointestinal
procedures. Other e ologies of SBO include malignant tumors (20%), hernias (10%), inammatory bowel
disease (5%), and volvulus (3%). From an emergency medicine standpoint, the most important steps are early
diagnosis and treatment. Left untreated, SBOs result in high mortality and morbidity rates. Patients with early
SBOs often present with nausea and diarrhea. As the obstruction progresses, the patient will likely start
vomiting, become febrile, and stop passing stool or flatus. Early SBOs can be missed at abdominal radiography;
therefore, if the suspicion is high, surgical consultation and CT scanning are mandatory.

3o clinical cases from eMedicine


Case 17
ABDOMINAL PAIN AND VOMITING IN A YOUNG GIRL
Background
A 2-year-old girl presents to the pediatric emergency department because of epigastric abdominal pain for the
last 3 days associated with fever; vomi ng; and watery, brown stools. Her parents deny hematemesis or blood
in her stool.
The episodic symptoms cause the patient to double over in pain and have been increasing in frequency, now
occurring every 20 minutes. The pa ent vomits a er every feeding, including liquid feedings, and her last bowel
movement occurred 24 hours prior to her admission. She was evaluated 2 days ago for similar complaints and
discharged home with a diagnosis of gastroenteritis.

Figure 1

The patient has a normal birth history, no medical problems, and no prior surgeries. Her immunizations are
current.On physical examina on, the girl's vital signs are as follows: rectal temperature, 100.2F; pulse, 134
beats per minute; respira ons, 32 breaths per minute; and oxygen satura on with pulse oximetry, 98% on room
air. The patient is alert and sitting comfortably with her parents. Her skin and mucous membranes are moist.
The remaining findings are normal and reveal a soft, nontender, nondistended abdomen without rebound or
guarding. Rectal examination is not performed.
While you speak to the parents, the patient clutches her abdomen and cries, doubling over in pain. The episode
spontaneously resolves after several minutes.
What is the diagnosis?
Hint
Note the episodic abdominal pain and positive radiographic findings.

3o clinical cases from eMedicine


Case 17 Answer
ABDOMINAL PAIN AND VOMITING IN A YOUNG GIRL
Diagnosis: Intussusception
Discussion:
The abdominal radiograph and barium enema study depict an ileocolic intussusception, which is demonstrated
by the acute intraluminal colonic filling defect.
Intussusception is a process in which a segment of intestine invaginates, or telescopes, into the adjoining
intestinal lumen, causing bowel obstruction. It is the most common cause of bowel obstruction in children
younger than 6 years. The classic triad of vomi ng, abdominal pain, and passage of currant jelly stools occurs in
less than one third of pa ents. Currant jelly stool occurs in 60% of cases and is caused by the obstruc on of
venous return (Wyllie, 2004).
If the intussusception is not reduced, the passage of bloody, mucoid stools; weakness; fever; and shock follows.
An upper respiratory infection often precedes these symptoms. The typical patient is usually a previously well
child presenting with recurrent severe paroxysmal colicky abdominal pain.
Correlations have been noted in patients with adenovirus infections and in infants receiving the tetravalent
rhesus-human rotavirus vaccine within 2 weeks of intussuscep on (odds ra o, 21:7) (Wyllie, 2004). Inamed
Peyer patches in the terminal ileum secondary to GI infection or the introduction of new food proteins are other
postulated mechanisms that might be responsible for creating the lead point in the telescoping process. Other
causes for lead points are polyps, Meckel diverticulum, neurofibroma, intestinal duplication, hemangioma,
malignancy, and cys c brosis. Lead points are most common in pa ents older than 2 years.
Postopera ve intussuscep on usually occurs within 5 days of an abdominal opera on. In rare circumstances,
the parents report 1 or more previous a acks of abdominal pain 10 days to 6 months prior to the current
episode. Strangula on of the bowel does not usually occur within the rst 24 hours of presenta on but may
occur later, leading to intestinal gangrene and shock.
The differential diagnosis includes malrotation of the bowel, chronic constipation, Hirschsprung disease,
appendicitis, inflammatory bowel disease, Henoch-Schnlein purpura, intestinal vascular malformations, peptic
ulcer with bleeding, and portal hypertension with bleeding varices.
Management consists of fluid resuscitation, placement of a nasogastric tube to minimize the risk of vomiting
and aspiration, preoperative laboratory studies, abdominal radiography, and surgical consultation. In the
absence of perforation, administering a barium or air enema can confirm the diagnosis and hopefully reduce
the intussusception. A surgeon must be ready to operate if this procedure is not successful. The patient should
be observed in the hospital for 24 hours or at home if the parents are reliable and if the pa ent can quickly
return if symptoms recur. The recurrence rate for an intussuscep on is 10% a er reduc on with barium enema
and 2-5% a er surgical resec on (Wyllie, 2004).
Radiographic findings are often normal in intussusception. Early radiographic evidence of an absence of air in
the right lower quadrant is known as the Dance sign (Hostetler, 2002). So -tissue opacity in the right upper
quadrant may be present in 25-60% of pa ents. Radiographic ndings illustra ng small
-bowel dilatation and airfluid levels may represent a small-bowel obstruction. If generalized distention with air-fluid levels is present in
the colon, acute gastroenteritis is the more likely diagnosis.

3o clinical cases from eMedicine


Case 18
SYNCOPE
Background
A 75-year-old man presents to the hospital with a cut on his forehead. He states that he must have fallen but
does not know why. He has a history of hypertension and diabetes mellitus.

Figure 1

Hint
What is the PR interval?

3o clinical cases from eMedicine


Case 18 Answer
SYNCOPE
Diagnosis: Third-degree atrioventricular (AV) block
Discussion:
This is a condition in which all supraventricular impulses are blocked at the AV junction and therefore
prevented from reaching and activating the ventricles. As a result, another intrinsic pacemaker activates the
ventricles. The supraventricular rhythm, which is usually faster, is completely disassociated from the slower
junc onal escape rhythm of less than 40 beats per minute or a slow idioventricular rhythm of less than 35 beats
per minute. Complete AV dissociation does not always result in complete AV block. Generally, in AV
dissociation, complete heart block is present if the atrial rate is greater than the ventricular rate, whereas heart
block is not present if the ventricular rate is greater than the atrial rate (eg, accelerated junctional rhythm,
accelerated idioventricular rhythm [AIVR], ventricular tachycardia [VT]).

3o clinical cases from eMedicine


Case 19
SYNCOPE AND HYPOTENSION
Background
A 48-year-old patient presents to the hospital with syncope and hypotension.

Figure 1

Hint
Note the S wave in lead I and the Q wave in lead III.

3o clinical cases from eMedicine


Case 19 Answer
SYNCOPE AND HYPOTENSION
Diagnosis: Acute pulmonary embolism
Discussion:
Sinus tachycardia, the most common ECG abnormality, and the classic S1Q3T3 pa ern are observed. Evidence
of right heart strain, a right bundle branch block (RBBB) pattern, and axis deviation is also present.

3o clinical cases from eMedicine


Case 20
RASH ON A CHILD'S LEGS
Background
One week ago, the mother of this 4-year-old girl noted small red dots on the child's legs, which the primary
pediatrician diagnosed as bug bites. The lesions became larger, and a schoolteacher confronted the mother
about what she thought were bruises on the child's legs. Concerned, the mother brought the child to a
dermatologist, and biopsy was performed; however, the mother states that no diagnosis was made. Now,
nearly hysterical, the mother brings her daughter to the emergency department and complains that the rash
looks worse and that the child refuses to walk. The girl is afebrile, appears well, and she is able to ambulate.

Figure 1

What is the diagnosis?


Hint
Children with this idiopathic syndrome often complain of abdominal pain. In some, intussusception or renal
dysfunction may even develop. When pressure is placed on the skin lesions, they are palpable but do not
blanch.

3o clinical cases from eMedicine


Case 20 Answer
RASH ON A CHILD 'S LEGS
Diagnosis: Henoch-Schonlein purpura (HSP), anaphylactoid purpura
Discussion:
The classic syndrome of HSP consists of a purpuric rash, arthritis, gastrointestinal symptoms, and renal
involvement. Skin lesions are required for the diagnosis and often appear as small wheals or erythematous
macules that progress to purpura. Angioedema of the face, hands, feet, and perineum is also common. Two
thirds of the patients have arthritis in the large joints, and more than half have gastrointestinal symptoms.
Colicky abdominal pain and vomiting are typical. Hemoccult results may be positive, or the stool can be grossly
bloody. The failure to recognize HSP can result in unnecessary laparotomy; however, intussusception or smallbowel obstruc on rarely complicates HSP. Approximately 25-50% of the pa ents have renal involvement.
Hematuria and proteinuria are the most common findings, but patients can have hypertension, azotemia,
oliguria, or nephrotic syndrome as well. Central nervous system involvement is extremely rare. Most patients
have a benign course and require only nonsteroidal anti-inflammatory drugs (NSAIDs) for symptomatic relief.
Relapses may occur. Patients with intussusception, small-bowel obstruction, or nephrotic syndrome are given
prednisone 1-2 mg/kg/d and require hospitaliza on.

3o clinical cases from eMedicine


Case 21
CHEST PAIN AT A PARTY
Background
A 32-year-old man presents to the hospital with chest pain that began at a party.

Figure 1

Hint
He is extremely young for atherosclerotic disease.

3o clinical cases from eMedicine


Case 21 Answer
CHEST PAIN AT A PARTY
Diagnosis: Acute anterior-septal myocardial infarction
Discussion:
Note the ST-segment eleva on in leads V1 through V4. Before his chest pain started, the pa ent was using
cocaine, which induced coronary vasospasm. This was his fifth such emergency department visit, all of which
had a similar clinical presentation.

3o clinical cases from eMedicine


Case 22
DEPRESSION IN A 65-YEAR-OLD WOMAN
Background
A 65-year-old white woman presents with depression, excessive sleepiness, chronic malaise, emotional lability,
constipation, muscular weakness, imbalanced gait, and arthralgias. Two years ago, after the death of her
husband of 45 years, she immigrated to the United States from Copenhagen, Denmark, to live with her son and
his family. Her medical, surgical, and family histories are normal except for approximately 1 tension headache
per month, which she successfully treats with 2 over-the-counter ibuprofen tablets. Her family physician has
treated her with uoxe ne (Prozac) for 8 months, without improvement in her symptoms.

Which of the following tests would be most helpful in the diagnosis of this case?
A.
B.
C.
D.
E.

Liver function tests


Complete blood count analysis
Thyroid-stimulating hormone (TSH) test
Urine toxicology screening
Rapid plasma reagin (RPR) test

Hint
The diagnosis is not idiopathic depression.

3o clinical cases from eMedicine


Case 22 Answer
DEPRESSION IN A 65-Y EAR -O LD WOMAN
Diagnosis: Hypothyroidism
Discussion:
The correct answer is C, a thyroid-stimulating hormone (TSH) test: This patient has depression and other
findings secondary to hypothyroidism. All of the tests listed are part of the diagnostic workup in patients in
whom depression is suspected. However, in this patient, the TSH measurement would be most helpful because
depression is common in hypothyroidism. This patient has spent most of her life in Denmark. Hypothyroidism is
much more common in Europe than in the United States, as a result of the prevalence of iodine deficiency
there. Several studies have shown that the prevalence of hypothyroidism in Europe may be as high as 2-5%. In
addition to depression, other neurologic sequelae can result from hypothyroidism. These include myopathy,
which caused this patient^s weakness, and neuropathy, which contributed to her imbalanced gait.

3o clinical cases from eMedicine


Case 23
A 20-YEAR-OLD SOLDIER WITH A 3-DAY HISTORY OF A DARK, BURNING RASH

Background
A 20-year-old active-duty male soldier presents to the emergency department (ED) of a military hospital
complaining of a 3-day history of a dark-red "burning rash." The rash started at his sock line and, over the
course of the past 2 days, has spread proximally up his thighs. It is not present on his abdomen or back, but it
has spread to his hands over the past day. The patient also developed a sore throat and a scratchy voice the day
before presentation, without odynophagia. He was in the North Carolina woods as part of his infantry training,
which is supplemental training after boot camp. He reports having spent 20+ hours per day for the past several
days in a foxhole (a hole in the ground that soldiers use for protection). The patient reports fatigue only related
to his level of activity and lack of sleep during training. He is not taking any medications, he has no allergies, and
he has smoked 5-10 cigare es daily for the past year. He also reports drinking approximately 5-10 beers with
his comrades 1-2 mes per month before star ng his training. The pa ent notes some right ankle swelling and
pain that started a er a 12.43 mi (20 km) eld hike with a 60 lb (27.22 kg) rucksack on his back. The pain and
swelling began 1 month before admission and has worsened in the past few days. He has had no pruritus,
fevers, chills, abdominal pain, diarrhea, changes in urine color, or dysuria, as well as no new sexual contacts and
no recent animal or insect bites. No one else in his military unit has reported similar skin findings. The patient
had a minor motorcycle accident 18 months ago, with a right leg lacera on that required suture repair. He has
no significant family history.

Figure 1

On physical examina on, his vital signs demonstrate a strong pulse, with a regular rhythm and a rate of 58 bpm,
blood pressure of 123/60 mm Hg, weight of 165.3 lb (75 kg), and oral temperature of 98.1F (36.7C). He is a
very fit, well-developed white male in no apparent distress. The pharynx shows no erythema or exudate, and
the neck examination demonstrates no tenderness to palpation or lymphadenopathy. The cardiac examination
is notable for a point of maximum intensity at the 5th le interspace, but nomurmurs, gallops, or rubs are
appreciated. The patient's pulses are strong bilaterally. The respiratory examination reveals lungs clear to
auscultation bilaterally. The abdominal examination is unremarkable, and the stool test is negative for occult
blood. The skin examina on show lesions ranging in diameter from 2 mm to 10 cm (see Figures 1-3). The
macules and plaques are nonblanchable, and most of them are concentrated in the posterior calves, the palms,

3o clinical cases from eMedicine


and the soles of the feet. No macules or plaques are noted on the face, chest, or back. The lesions are not
tender to palpa on or warm to the touch. His hands and right ankle exhibit 1+ nonpi ng edema. The pa ent
has a scar on his right leg from his motorcycle accident that is well-healed, with no fluctuance or erythema. The
neurologic examination is unremarkable.

Figure 2

A urinalysis shows 2+ protein and 2+ blood, with 25-50 red blood cells (RBCs) per high-power field on
microscopy. His coagulation studies at admission include a prothrombin me (PT) of 12.3 s, an interna onal
normalized ra on (INR) of 1.08, and a par al thromboplas n me (PTT) of 25.5 s. The pa ent's blood urea
nitrogen (BUN) is 24 mg/dL (8.59 mmol/L) and his crea nine value is 1.1 mg/dL (97.2mol/L). The complete
blood count (CBC) shows a white blood cell (WBC) count of 7.4 103/L (7.4 109/L), a hemoglobin (HGB) of
12.4 g/dL (124 g/L), a hematocrit (HCT) of 37.1% (0.371), and a platelet count of 195 103/L (195 109/L), with
a normal smear. Skin punch biopsies of 2 of the lesions are obtained that demonstrate small-vessel
leukocytoclastic vasculitis. Immunofluorescence of the skin biopsies demonstrates a weak linear pattern at the
dermal-epidermal junction for immunoglobulin G (IgG), immunoglobulin A (IgA), and complement 3 (C3).

Figure 3

3o clinical cases from eMedicine


What is the diagnosis?
Hint: The patient is presenting with nonblanchable lesions after reporting an antecedent sore throat.
o
o
o
o
o

Henoch-Schnlein purpura (HSP)


Mixed cryoglobulinemia
Polyarteritis nodosum
Systemic lupus erythematosus
Hemolytic uremic syndrome (HUS)

3oo clinical cases from eMedicine


Case 23 Answer
A 20-YEAR-OLD SOLDIER WITH A 3-DAY HISTORY OF A DARK, BURNING RASH

Diagnosis: Henoch-Schnlein
Schnlein purpura (HSP)
Discussion:
There exist several guidelines for the diagnosis of Henoch-Schnlein
Henoch Schnlein purpura. The American College of
Rheumatology (ACR) requires
es 4 criteria (published in 1990) for diagnosing HenochHenoch-Schnlein purpura: palpable
purpura, pa ent age of 20 years or less at onset, bowel angina, and the presence of granulocytes in the vessel
walls. More recently, in 2006, the European League Against Rheumatism
Rheumatism (EULAR) and the Pediatric
Rheumatology Society published their own Henoch-Schnlein
Henoch Schnlein purpura criteria. These include palpable purpura
as a mandatory criterion, with at least one of the following conditions: diffuse abdominal pain, predominant IgA
deposition (confirmed on biopsy), acute arthritis in any joint, and renal involvement (as evidenced by hematuria
and/or proteinuria). According to the older ACR criteria, the diagnosis of Henoch
Henoch-Schnlein purpura for our
patient could be called into question,
on, as he exhibited no bowel angina; however, according to the more recent
EULAR criteria, our patient fits the diagnosis of Henoch-Schnlein
Henoch Schnlein purpura quite well, as he has palpable
purpura, predominant IgA deposition confirmed on biopsy, a monoarticular arthralgia
arthralgia in his right ankle, and
renal involvement.
The sine qua non for the diagnosis of Henoch-Schnlein
Henoch Schnlein purpura is a skin biopsy finding of IgA deposition at the
dermal-epidermal
epidermal junction. A skin biopsy with immunofluorescence studies is recommended if clinical suspicion
of Henoch-Schnlein
Schnlein purpura exists. The etiology of the disease remains unknown, but it is understood to be an
autoimmune response (usually to an upper respiratory infection).
Other items in the differential diagnosis include many of the etiologies of palpable purpura. An algorithmic
a empt has been made (see Table 1) to delineate these e ologies, but classica on remains dicult.Palpable
purpura is a classic skin manifestation of cutaneous vasculitis, but there are many other entit
entities that can cause
it. Additional information can be gleaned from the pathologic diagnosis of small vessel vasculitis, which also
carries its own dieren al (see Table 2). The intersec on of these 2 dieren als consists of the following set of
diagnoses:
es: mixed cryoglobulinemia, vasculitis associated with collagen vascular disease (ie, systemic lupus
erythematosus, rheumatoid arthritis), and Henoch-Schnlein
Henoch
purpura.
Mixed cryoglobulinemia was easily ruled out, as the patient's serum cryoglobulin test aand complement levels
were a C3 of 118 mg/dL (1.18 g/L; normal range, 79-152
79 152 mg/dL), a C4 of 26 mg/dL (0.26 g/L; normal range, 16
1638 mg/dL), complement CH50 of 53.0 U/mL (53 kU/L; normal range 22-60
22 60 U/mL), and a nega ve serum
cryoglobulin examination. Another
her diagnosis that was readily excluded was systemic lupus erythematosus. Our
pa ent had only 1 arthralgia, but no fevers, myalgia, or malaise were noted; only 1/11 American Rheumatologic
Association criteria for the diagnosis of lupus were met. To satisfy
satisfy the clinical diagnosis of systemic lupus
erythematosus, 4/11 American Rheumatologic Associa on criteria are required. Although the pa ent meets the
histopathologic criteria for solitary IgA nephropathy, as the renal biopsy showed diffuse proliferative
endocapillary glomerulonephritis with increased mesangial matrix and cellularity, capillary wall thickening, and
lobular accentua on on light and electron microscopy with strong (3+) mesangial staining for IgA and
fibrinogen, as well as lappa and lambda light
l
chains (2-3+)
3+) present on immunouorescence microscopy, he also
exhibited some of the cardinal clinical features of Henoch-Schnlein
Henoch Schnlein purpura at presentation. He denied having

3o clinical cases from eMedicine


had blood in his urine on prior urinalyses or a history of gross blood urine, lessening the possibility of IgA
nephropathy. After further inquiry, it was confirmed that the patient had no family history of renal disease and,
as such, it is unlikely that he has familial IgA nephropathy. Polyarteritis nodosum (PAN) could be considered in
the differential diagnosis, but this patient displayed none of the classic signs or symptoms of fever, abdominal
pain, nausea, and weight loss. Also common to PAN is the is involvement of small- and medium-sized arteries,
as well as subcutaneous nodules (not palpable purpura). Joint pain, however, is very common in PAN. PAN may
be associated with hepatitis B or C, but this patient's hepatitis panel was negative. A hemorrhagic diathesis was
quickly ruled out with the normal coagulation studies. Lyme disease, in contrast to Henoch-Schnlein purpura,
more commonly presents with the typical target lesions of erythema migrans and a positive Lyme titer (and not
with a purpuric rash). This patient had a negative Lyme antibody examination, although this finding alone does
not exclude the possibility of the disease. Multiple arthralgias are more common in Lyme disease, and they
seldom present with abdominal pain. Hemolytic uremic syndrome (HUS) should be considered in the differential
diagnosis, but it can be easily recognized by the presence of microangiopathic anemia and is distinguished from
Henoch-Schnlein purpura by the absence of purpura and arthralgias.
Henoch-Schnlein purpura is an autoimmune, self-limited IgA-mediated vasculitis that most commonly presents
in children 5 years of age. In fact, 95% of cases present in children <10 years of age. There is a 2:1 distribu on
between males and females. The overall incidence is 0.14 cases per 1000 popula on in children aged 2-14. This
patient is in a relatively uncommon demographic for the disease. The most common signs and symptoms of the
disease are palpable purpura (100%), abdominal pain (63%), gastrointes nal bleeding (33%), arthralgia (82%),
nephri s (40%), and hematuria. Risk factors associated with apoor prognosis for renal survival include
nephrotic syndrome, decreased factor XIII activity, hypertension, and renal failure at onset. It is known that
severe Henoch-Schnlein purpuraassociated nephritis and progression to nephrotic syndrome are associated
with a poorer prognosis.
The mainstays of treatment for Henoch-Schnlein purpura are observation, intravenous hydration, and steroid
therapy. Most cases are self-limited and resolve within 4 weeks. Cyclophosphamide is usually reserved for
severe, systemic forms of vasculitis, and appropriate consultation with a nephrologist is recommended. Chronic
kidney disease and end-stage renal disease are known complications of Henoch-Schnlein purpura, with
incidence rates of 5% and 1.5%, respec vely. It should be noted that corticosteroids should only be instituted
a er consulta on with a nephrologist, as they are of ques onable benet. In fact, only 1 randomized controlled
clinical trial has been performed evaluating the use of corticosteroids in treating Henoch-Schnlein purpura as
renal protective agents. Huber et al found that early prednisone therapy initiation did not ward off chronic
kidney disease at 1 year; however, there has been a case report of successful treatment of an adult pa ent with
crescentic glomerulonephritis (from Henoch-Schnlein purpura) with 9 rounds of double-filtration
plasmapheresis and concomitant plasma replacement.
Our patient was admitted to the hospital ward and vigorously volume-expanded with normal saline at 250 cc/h.
His crea nine clearance was 182.7 mL/min (3.05 mL/s; this is within normal limits). Also notable was his
an streptolysin O (ASO) ter of 722 Todd U/mL (normal range,-117
0 Todd U/mL). Even with his severe skin
manifestations, the severity of our patient's Henoch-Schnlein purpura nephritis on admission was low. The
patient's positive ASO titer pointed to a recent presumed streptococcal pharyngitis. The skin biopsy report was
significant for small-vessel leukocytoclastic vasculitis. The immunofluorescence study of the sample
demonstrated a weak linear pattern at the dermal-epidermal junc on, with IgG, IgA, and C3, which was
consistent with the diagnosis of Henoch-Schnlein purpura. On admission, the patient's protein-to-creatinine
ra o was 5.5, consistentwith nephrotic-range proteinuria, or >3.5 grams of protein per day. A normal proteinto-crea nine ra o is <0.2. Although the nephrology service was consulted early in the clinical course and the
patient was started on high dose intravenous methylprednisolone therapy at 1 gram divided twice a day on
hospital day 1, his condi on rapidly worsened. By hospital day 4, the pa ent's serum crea nine was 1.1 mg/dL
(97.2 mol/L) and his protein-to-crea nine ra o had increased to 15.1. His urine volume was 1850
mL on that
day, with 24 h protein and crea nine excre on totaling 35.8 grams and 2384 mg, respec vely. The pa ent,
while still normotensive, had developed prominent edema consistent with nephrotic syndrome. As his renal
status had worsened significantly, he was transferred to a tertiary care center for further nephrology care. A
subsequent renal biopsy demonstrated diffuse proliferative endocapillary glomerulonephritis consistent with
IgA nephropathy, or Henoch-Schnlein purpura. Mild chronic tubulointerstitial disease and mild hyaline

3o clinical cases from eMedicine


arteriolar sclerosis were noted as well. A regimen of cyclophosphamide was initiated, and the patient
deteriorated to end-stage renal disease by the time this case was compiled.

Table 1: E

ology of Purpura

Intravascular (usually nonpalpable)

Vascular (usually palpable)

Alterations in platelet formation, destruction, or function


Drugs (aspirin, methyldopa)
Idiopathic thrombocytopenic purpura
Thrombotic thrombocytopenic purpura
Disseminated intravascular coagulation
Infection (especially Neisseria, Rickettsia, Staphylococcus)
Splenic sequestration
Radiation therapy
Myelofibrosis
Myeloproliferative disorders

Inflammatory vascular causes

Cutaneous vasculitis
Purely cutaneous vasculitis (eg, secondary to medications)
Henoch-Schnlein purpura
Polyarteritis nodosa
Granulomatous vasculitis (Wegener granulomatosis,
Churg-Strauss vasculitis)
Cutaneous vasculitis associated with a collagen
Vascular disease (eg, systemic lupus erythematosus,
rheumatoid arthritis)
Giant cell arteritis
Mixed cryoglobulinemia
Hyperglobulinemic purpura

Noninflammatory vascular causes

Extravascular and miscellaneous (may be


palpable or nonpalpable)

Trauma
Subacute bacterial endocarditis
Other embolic diseases
Amyloidosis

Corticosteroids
Toxins and venoms
Senile purpura
Scurvy
Valsalva maneuver
Pseudopurpura (Sweet syndrome, cherry angiomas,
angiokeratoma, Kaposi sarcoma)

3o clinical cases from eMedicine


Table 2: Clinical Signs of Necro

zing Vasculi

s with Respect to Vessel Size Involved

Small Vessels
Signs

Diseases

Urticaria

Hypersensitivity vasculitis

Palpable Purpura

Henoch-Schnlein purpura
Essential Mixed Cryoglobulinemia
Vasculitis associated with connective tissue disease
Vasculitis associated with malignancy
Serum sickness and serum sickness-like reactions

Nodules, bullae, or ulcers

Chronic urticaria (urticarial vasculitis)


Urticarial prodrome of acute hepatitis type B infection

Large Vessels
Signs
Subcutaneous
ecchymoses

Diseases
nodules,

ulceration,

and,

Polyarteritis nodosa
Churg-Strauss syndrome
Wegener granulomatosis
Giant cell (temporal) arteritis

3o clinical cases from eMedicine


Case 24
A TRAVELERS FEVER
Background
A 17-year-old man presents to the emergency department (ED) complaining of a headache that has lasted for
the past 2 days. The pa ent states that he returned to the United States 2 days ago a er spending 3 weeks in
Nigeria. According to the patient, he felt well when he initially arrived in the United States, but he developed a
severe headache soon after. The headache is constant and throbbing, is lasting throughout the day, and is
relieved with ibuprofen. He complains of subjective fevers and intermittent sweats, especially at night, but he
has not taken his temperature.

Figure 1

Today, while preparing to board a plane, he developed a worsening headache, bilious emesis, palpitations, and
sweats. He decided to delay his trip and is now presenting to the local ED for further evaluation. The patient
denies having any trauma, seizures, abdominal pain, stiff neck, or photophobia. He has no significant past
medical history, and his only medica on use is the ibuprofen that he has taken over the past 2 days.
On physical examina on, his temperature is 103.0 F (39.4 C), his pulse is 83 bpm, his blood pressure is 120/70
mm Hg, his respiratory rate is 16 breaths/min, and his oxygen satura on is 96% while breathing room air. The
patient is generally well-appearing, alert, and oriented. The examination of the head, eyes, and pupils is
unremarkable. The neck is supple, without any lymphadenopathy. On auscultation, the lungs are clear;
additionally, the patient's heart has a regular rate and rhythm, without any murmurs. The examination of the
abdomen reveals normal bowel sounds, with mild tenderness to palpation in the right and left upper quadrants.
The spleen is noted to be 3 cm below the costal margin. The neurologic examina on is unremarkable.

3o clinical cases from eMedicine


The patient is treated with intravenous fluids. An electrocardiogram (ECG) shows a normal sinus rhythm, with a
QTc of 390 msec. Serum laboratory tests are done, and they are signicant for the following values: crea nine,
1.2 mg/dL (106.08 mol/L); magnesium, 1.2 mg/dL (0.49 mmol/L); alanine aminotransferase (ALT), 110 U/L
(normal range 0-35 U/L); aspartate aminotransferase (AST), 159 U/L (normal range 0-35 U/L); total bilirubin, 2.7
mg/dL (46.17 mol/L) (normal range <1.20 mg/dL); direct bilirubin, 0.9 mg/dL (15.39 mol/L) (normal range
<0.20 mg/dL); and alkaline phosphatase, 152 U/L (normal range 30-130 U/L). A complete blood count (CBC)
reveals a white blood cell (WBC) count of 5.7 103/L (5.7 109/L), a hematocrit of 44 % (0.44), and platelets of
34 103/L (34 109/L). A blood smear is obtained (see Figure 1).
What is the diagnosis?
Hint: Pay close attention to the finding of the blood smear.
o
o
o
o

Malaria
Babesiosis
Lyme Disease
Ehrlichiosis

3oo clinical cases from eMedicine


Case 24 Answer
A TRAVELERS FEVER

Diagnosis: Malaria
Discussion:
This case is an example of malaria caused by Plasmodium falciparum.. In the context of the patient's recent
travel to Nigeria and presentation with fever, malaria was strongly suspected. The patient's blood smear was
notable for 15% parasitemia, most likely P falciparum (as evidenced by the circles within some of the red blood
cells [RBCs] in the smear seen in Figure 2). The other laboratory ndings, such as the thrombocytopenia and
elevated bilirubin, correlated with the disease burden. The elevated creatinine
creatinine value was also noted, as this
finding is sometimes seen in more aggressive cases of the disease; however, this usually resolves with time.
Worldwide, there are about 300-500
500 million new cases of malaria annually. Malaria is the most deadly vector
vectorborne illness in the world, causing 3.5-5
3.5 5 million deaths annually. On average, 40% of the 50 million people who
travel from industrialized to developing
developing countries each year report some illness associated with their travel.
Approximately 1,200 cases of malaria are reported each year in the United States among travelers; therefore, it
is important to consider malaria as a possible cause of fever in the returning traveler.
Malaria results from an infec on caused by any of the following 4 protozoa of the genusPlasmodium:
falciparum, vivax, ovale and malariae.
malariae. Transmission of the parasite occurs via the bite of the Anopheles
mosquito. Once the protozoa are
re injected into the bloodstream, they enter the hepatic cells and reproduce;
eventually, the hepatic cells erupt and release more protozoa into the host's circulation. These parasites then
remain in the bloodstream, periodically invading erythrocytes, causing
causing hemolysis, and infecting new RBCs.
The incuba on period tends to be 9-18 days for P falciparum, P vivax and P ovale
ovale, but P malariae has an
incuba on period of 18-40 days. The most common parasites seen in the US are P falciparum, which is often
found
d in travelers returning from Sub-Saharan
Sub
Africa, and P vivax,, which is found in those returning from Asia,
Eastern Europe, and La n America. The clinical presenta on also varies between these 2 parasites:
P falciparum
often causes symptoms within the first
first month following the travel period, and it can be fatal; of patients
infected with P vivax,, 50% have symptoms within 1 month a er travel, and approximately 2% of pa ents may
have symptoms 1 year a er exposure. The majority of pa ents infected with ither
e
parasite are usually
symptoma c within the rst 3 months a er they return to the US.
The clinical presentation of malaria can vary widely and depends on the species of Plasmodium involved.
Common symptoms include fever, malaise, myalgias, and headache,
headache, which may be accompanied by cough,
abdominal pain, or diarrhea. Since these symptoms are non-specific,
non specific, malaria should be considered in all febrile
travelers, regardless of their clinical presenta on. In fact, approximately 78-100% of pa ents presenting with
malaria are febrile when they are first examined. The classically described fever patterns are rarely observed;
however, when these fevers do occur at 4848 to 72-hour
hour intervals, this finding is virtually pathognomonic for P
vivax, P ovale, and P malariae infections. This cyclical pattern of symptoms coincides with the regular interval of
erythrocyte hemolysis. On examina on, splenomegaly is found in 24-48% of pa ents, and pa ents may
complain of abdominal pain. Severe malaria, usually caused by P falciparum,, causes several manifestations,
including prostration, impaired consciousness or coma, respiratory distress caused by pulmonary edema and
acute respiratory distress syndrome (ARDS), seizures, circulatory collapse, abnormal bleeding (including

3o clinical cases from eMedicine


disseminated intravascular coagulopathy), splenic rupture, jaundice, severe anemia, acute renal failure, and
acidosis. The level of parasitemia o en exceeds 5%.
The diagnosis of malaria is made by examination of both thin and thick blood smears. These smears are used to
quantify the level of parasitemia, which is used to guide treatment. If the first smear is negative, it should be
repeated at 12- to 24-hour intervals for 48-72 hours. If the diagnosis is clinically suspected and a sucient
laboratory diagnosis is not possible, empirical treatment for P falciparum should be started because the disease
can be fatal if left untreated. Other laboratory findings, such as normocytic anemia, thrombocytopenia, low
WBC count, elevated lactate dehydrogenase, and elevated bilirubin, are nonspecific, but they may provide clues
to the diagnosis.
The treatment options for malaria vary according to the region where the traveler was likely to have acquired
the infection. This is based on the local prevalence and antimalarial drug susceptibility of particular Plasmodium
species. Chloroquine is the treatment of choice for those patients infected with P vivax, P ovale, and P malariae.
Patients infected with either P vivax or P ovale must also take primaquine, as these species may lie dormant
within the liver and are not eradicated by chloroquine alone. Failure to treat P vivax and P ovale with both
agents frequently leads to recrudescence of the disease. If P falciparum has been eliminated as the causative
agent, patients may be treated in the ambulatory setting. If P falciparum is suspected, treatment should be
ini ated as soon as possible, as severe illness or death can occur in as li le as-21 days a er presenta n.o The
CDC recommends hospitalization in order to ensure that patients are able to tolerate oral therapy and have an
appropriate response to treatment. For those patients returning from areas where Plasmodium species have
known chloroquine resistance, such as Southern Asia, Sub-Saharan Africa, and Oceania, there are 3 available
treatment op ons: (1) oral quinine along with tetracycline, doxycycline, or clindamycin; (2) atovaquone
proguanil; or (3) meoquine along with doxycycline. Depending on the severity of the disease and the region
where the parasite was acquired, dierent lengths of treatment are recommended; these vary from 3 to 7 days
of quinine therapy and 7 days of an bio cs. For those pa ents with severe malaria, an ini al course of
doxycycline with quinidine, an intravenous isomer of quinine, is recommended un l the parasite burden is <1%,
at which point oral therapy should be initiated. Unfortunately, quinine and quinidine have several side effects,
and patients must be closely monitored for cardiac dysrhythmias caused by prolongation of the QTc, profound
asymptomatic hypoglycemia, and respiratory distress.
In this case, the diagnosis of malaria resulting from P falciparum infection was promptly made based on the
patient's travel history and the severity of his symptoms. He was admitted to the pediatric intensive care unit
for close observation because of the potentially dangerous side effects of treatment. The patient received a
bolus of intravenous quinidine and clindamycin. His parasitemia was <0.5% on the second day of treatment, and
the patient was switched to oral quinine therapy and clindamycin. His platelet and creatinine levels improved
on hospital day 5, and he was discharged to home on oral therapy for a total of 7 days.

3o clinical cases from eMedicine


Case 25
ACUTE ONSET OF ABDOMINAL PAIN IN A 76-YEAR-OLD MAN
Background
A 76-year-old man presents to the emergency department (ED) complaining of a sudden onset of abdominal
pain.

Figure 1

The pain started about 4 hours before presenta on to the EDand has been persistent; it is present in the upper
abdomen and is centered in the epigastrium. He describes the pain as deep and burning. There is no associated
nausea or vomiting. He does not report any changes in his bowel habits and has not experienced any recent
fevers. The review of systems is also negative for any recent unintended weight loss or trauma. The patient also
reports having had "indigestion" in the past that caused pain similar to what he is currently experiencing,
though much less in intensity. His past medical history is significant for coronary artery disease and
hypertension. He takes two medications, both for his high blood pressure, but does not drink excessively and
does not smoke.
On physical examination, the patient is pale and in obvious severe discomfort. His heart rate is 122 bpm, and his
blood pressure is 110/65 mm Hg. He is breathing with rapid shallow breaths at a rate greater than 30
breaths/min. His temperature is normal at 99.2F (37.3C), and a pulse oximetry reading while the patient is
breathing room air shows a satura on rate of 100%. The cardiovascular and respiratory ndings are
unremarkable. The patient has significant tenderness in the epigastric region, with a rigid abdomen. There is
little to no tenderness to palpation in the lower quadrants; a reliable assessment of the upper quadrants is not
possible because of the tenderness in the epigastric region. Hyperactive bowel sounds are heard on
auscultation. The patient's stool is brown and guaiac positive.
An electrocardiogram is performed and is noted to be unremarkable except for sinus tachycardia. A complete
blood count (CBC) and a chemistry panel are sent. The CBC reveals a mild anemia, with a hemoglobin
concentra on of 127 g/L (12.7 g/dL). On the chemistry panel, there is evidence of a slight azotemia, with a
blood urea nitrogen level (BUN) of 17.1 mmol/L (48 mg/dL) and a crea nine value of 106 mol/L (1.2 mg/dL).
The remainder of the laboratory investigations are unremarkable. Plain radiographs of the abdomen are
performed (see Figures 1A and 1B).

3o clinical cases from eMedicine


What is the diagnosis?
Hint: Both the inner and the outer walls of the bowel are visible.
o
o
o
o

Abdominal aortic aneurysm rupture


Pancreatitis
Pneumoperitoneum from duodenal ulcer perforation
Acute coronary syndrome

3oo clinical cases from eMedicine


Case 25 Answer
ACUTE ONSET OF ABDOMINAL PAIN IN A 76-YEAR -OLD MAN

Diagnosis: Pneumoperitoneum from duodenal ulcer perforation


Discussion:
Clear deni on of both the inner wall and outer wall of the bowel on Figure 2A (the Rigler sign; see below for
explanation)
n) and the presence of free air under the right hemidiaphragm on Figure 2B demonstrate
pneumoperitoneum. The term "pneumoperitoneum" refers to air in the peritoneal cavity. The differential
diagnosis of pneumoperitoneum includes iatrogenic causes (eg, peritoneal
peritoneal dialysis; abdominal surgery,
including laparoscopy and laparotomy, a er which pneumoperitoneum can persist for up to 28 days), blunt or
penetrating trauma, perforation of the hollow viscera (eg, gastric ulcer, duodenal ulcer), pneumatosis
intestinalis
lis or pneumatosis coli, vaginal insufflation, and gas from the mediastinum (eg, from barotrauma).
Although many patients with a perforated peptic ulcer initially present with severe abdominal pain as well as
epigastric tenderness and classic signs of peritonitis,
peritonitis, patients who are elderly, are immunocompromised, or
have an altered mental status may have only minimal signs and symptoms. In one study of pa ents >60 years
with perforated ulcers, only 70% had abdominal pain. Other pa ents reported symptoms that included
dyspepsia, anorexia, nausea, and vomi ng. Severe abdominal pain was present in only 16% of pa ents. About
6% of pa ents with perforated ulcers have no abdominal ndings. In most cases of perfora on from ulcer
disease, gastric and duodenal content (ie, bile, ingested food, swallowed bacteria) leaks into the peritoneum,
resulting in peritonitis and an increased risk of infection and abscess formation. In addition to sepsis,
subsequent third-spacing
spacing of fluid into the peritoneal cavity caused by
by the peritonitis can lead to hypotension
and shock.
The underlying pathophysiology of a duodenal ulcer is a common condition that is characterized by the
presence of a well-demarcated
demarcated mucosal defect in the duodenum. Approximately 95% of duodenal ulcers ar
are
found in the rst part of the duodenum; most are less than 1 cm in diameter. A prompt and accurate diagnosis
combined with treatment can effectively prevent potentially serious complications, such as perforation (which
occurred in this case). The duodenal
al mucosa resists damage from the effects of gastric acid and the proteolytic
enzyme pepsin as a result of the protective qualities of the mucous/gel layer produced by the mucus
mucus-secreting
epithelial cells, bicarbonate secretions from other gastric and duodenal
duodenal cellular components, and protective
prostaglandins. If gastric acid and pepsin penetrate the mucous layer and reach the epithelial cells, ion pumps in
the basolateral cell membrane regulate intracellular pH by removing excess hydrogen ions; healthy ep
epithelial
cells migrate to the site of the injury; and mucosal blood flow serves to remove any excess acid diffused through
the injured mucosa. Despite these barriers and mechanisms to prevent permanent injury, ulcerations can occur.
Any pathophysiologic or iatrogenic process that increases gastric acidity (eg, disease states with increased
maximal and basal acid output), decreases prostaglandin production (eg, nonsteroidal anti
anti-inflammatory drug
[NSAID] use, which leads to inhibition of the cyclooxygenase-1
cyclooxygenase [COX-1]
1] pathway), or interferes with the mucous
layer (eg, Helicobacter pylori infection leading to stimulation of gastric acid production) can result in the
formation of peptic ulcer disease.
In the US, the prevalence of duodenal ulcer is es mated to be 6-15% in the general popula on. The majority of
individuals with duodenal ulcers do not have clinically significant disease. The prevalence is closely linked to the
presence of H pylori infection.
ion. Of those individuals infected with H pylori,, the lifetime prevalence is

3o clinical cases from eMedicine


approximately 20%. Interna onally, the prevalence of the disease varies among countries and, as in the US, is
linked to rates of H pylori infection.
Duodenal ulcers can result in significant morbidity and mortality. The main complication from these ulcers is
pain; however, serious complications can include ulcer hemorrhage, perforation, penetration, and obstruction.
Complications and mortality are generally greater in elderly patients than in other patient populations, possibly
because of a higher incidence of comorbid disease and an increased use of NSAIDs in this group.
When a perforation is suspected, as in this case, an upright chest or left lateral decubitus radiograph can detect
as li le as 1-2 mL of gas under the diaphragm or lateral margin of the liver, especially if strict posi oning
techniques are used. Therefore, these studies should be the first diagnostic modalities used. Supine abdominal
radiographs are generally of limited value in diagnosing pneumoperitoneum. Computed tomography (CT)
scanning is highly sensitive for depicting pneumoperitoneum, and this modality has the added benefit of
assisting the clinician in identifying the underlying etiology in many patients.
Numerous signs are described for diagnosing pneumoperitoneum on plain radiographs. One of the best known,
the Rigler sign (also known as the double-wall or bas-relief sign), is a visualization of the outer surface of a
bowel loop wall resulting from free air in the peritoneal cavity. The intraluminal gas provides negative contrast
and outlines the internal wall. The cupola sign, typically seen on supine radiographs, is an inverted cup-shaped
arcuate lucency overlying the lower thoracic spine and projecting caudally to the heart. This sign is formed as air
accumulates anteriorly in the median subphrenic space under the central leaf of the diaphragm. The umbilical
ligaments, the urachus, and particularly the falciform ligament are sometimes identified as linear radiopaque
structures in the presence of free air.
Another common sign is a collection of gas in the right upper quadrant adjacent to the liver, lying mainly in the
subhepatic space and in the hepatorenal fossa, that is visible as an oval or triangular gas shadow not in obvious
continuity with the rest of the bowel. This collection is usually present in the medial aspect of the right upper
quadrant, with a superomedial to inferolateral orientation.
The football sign is visualization of the entire peritoneal cavity as an oval gas shadow, with the vertically
oriented falciform ligament representing the seam of an American football. This sign is most often seen in the
pediatric patient but not in the adult patient, because in the adult patient, there is usually not enough air
relative to the size of the peritoneal cavity.
Nonsurgical management of a perforated ulcer is associated with prohibitive morbidity and mortality rates,
especially in high-risk groups, such as immunocompromised patients and the elderly. Surgical management is
generally indicated. Initial management includes gastric decompression with a nasogastric tube, pain control,
intravenous hydration, and administration of broad-spectrum antibiotics. Closure with a piece of omentum
(Graham patch) and truncal vagotomy with pyloroplasty (by incorpora ng the perfora on) are 2 common
approaches to the surgical management of a perforated duodenal ulcer. The patient in this case underwent an
omental patch repair for the duodenal perforation.

3o clinical cases from eMedicine


Case 26
AN 8-DAY-OLD BOY WITH TACHYPNEA AND DIFFICULTY FEEDING
Background
An 8-day-old boy is brought into the emergency department (ED) by his parents for a chief complaint of rapid
breathing. During the past several hours, the patient's mother noted an increased respiratory rate and
decreased feeding ability. The child had previously fed for 10 minutes at each breast every 2 hours, but he is
now crying and unable to latch on for more than a few seconds at each breast. Both parents state that they
think the patient looks pale. Up until the day of presentation, the patient had been doing well, with no prior
episodes of difficulty breathing. He was born full-term via normal spontaneous vaginal delivery, without
complications during the pregnancy or delivery. He has had no sick contacts and has not been in day care. He
has not had a fever, cough, or runny nose. His urine output has been normal, and he has had 2-3 nonbloody
bowel movements daily. Other than spitting up a small amount after feedings, there have not been any
episodes of vomiting. The parents state that he seems increasingly fussy. There have been no episodes of
apnea, cyanosis, choking, or gagging, as well as no discernible changes in muscle tone.

Figure 1

On physical examination, his vital signs are signicant for a respiratory rate of 70 breaths/min and a heart rate
of 296 bpm. He is afebrile, with a rectal temperature of 98.2F (36.8C). His blood pressure is 72/40 mm Hg. His
oxygen satura on, measured by pulse oximetry, is 98% while breathing room air. Despite the abnormal vital
signs, the baby does not appear distressed or even uncomfortable. He is moving all extremities, and he opens
his eyes and looks around. His oropharynx is clear, with no visible foreign bodies. There is no rhinorrhea or nasal
congestion. His lung sounds are clear, and despite the significant tachypnea, no retractions, grunting, or nasal
flaring are present. On auscultation of the heart, rapid heart sounds are noted, and as a result of the
tachycardia, an assessment for murmurs is not possible. His distal extremities are pink and have a normal
capillary refill.While placing an intravenous line and connecting the child to a monitor, an electrocardiogram
(EKG) is obtained.

3o clinical cases from eMedicine

What type of arrhythmia is shown on the EKG?


Hint: Consider the morphology of the QRS complexes (narrow vs wide) and the timing of the complexes (regular
vs irregular).
o
o
o
o

Torsade de pointes
Atrial fibrillation
Supraventricular tachycardia
Ventricular tachycardia

3oo clinical cases from eMedicine


Case 26 Answer
AN 8-DAY -OLD BOY WITH T ACHYPNEA AND DIFFICULTY F EEDING

Diagnosis:Supraventricular
Supraventricular tachycardia
Discussion:
This patient was diagnosed with supraventricular tachycardia (SVT), which is defined as a regular, rapid rhythm
that requires only atrial or atrioventricular tissue for its initiation and maintenance. Paroxysmal SVT (PSVT) is
the most common dysrhythmia in children. It has an interna onal prevalence of about 2 cases per 1,000 people.
PSVT tends to manifest in infancy and early childhood.
The presentation
resentation of PSVT is quite variable in children, ranging from an incidental finding in an asymptomatic
patient to fulminant cardiogenic shock. Infants present with caretakers complaining of rapid breathing, poor
feeding, sweating with feeding, pallor, lethargy,
lethargy, and excessive crying. Older children may complain of chest
pain, shortness of breath, and palpitations. The physical examination is likewise variable, depending upon the
child's age and heart rate and on the duration of the episode (which can last from seconds to days). For infants,
the examination is remarkable for a regular tachycardia. Normal resting heart rates for neonates can be up to
160 beats per minute. The upper limit of the normal heart rate decreases with age un l late adolescence, when
children's heart rates are similar to those of adults. Heart rates ranging beyond the normal upper limits,
par cularly in excess of 240, should be highly suspicious for SVT. In addi on to tachycardia, infants may have
physical findings of pallor, irritability,
bility, lethargy, tachypnea, weight loss (or failure to gain), poor perfusion, weak
pulses/hypotension, hepatomegaly, and, sometimes, cardiogenic shock. A pounding sensation in the neck may
be caused by cannon A waves, which occur when the atrium contracts at the same time as the ventricle. Older
children typically have benign examinations (except for the findings of tachycardia and tachypnea).
There are 3 types of SVT: (1) atrial tachycardia (ectopic, or nonreciproca ng, atrial tachycardia), (2)
atrioventricular
ricular nodal reentrant tachycardia (AVNRT), and (3) atrioventricular reentrant (or reciproca ng)
tachycardia (AVRT). In the United States, reentrant tachycardias are the most common cause of PSVT in the
pediatric population, with AVRT being more common than
than AVNRT. AVRT consists of 2 or more func onally (and,
usually, anatomically) distinct pathways between the atria and ventricles. The first pathway is usually the
atrioventricular (AV) node. The second is an accessory pathway that may be an anatomically separate bypass
tract between the atrium and ventricle (such as the bundle of Kent). Wolff-Parkinson
Wolff Parkinson-White (WPW) syndrome
preexcitation is a good example of SVT caused by an anatomically separate bypass tract. Each pathway has
different electrophysiologic characteristics; one pathway is fast (a short conduction time and a long refractory
period), while the other is slow (a longer conduction time and a shorter refractory period). In AVNRT, both
pathways exist in the AV node itself. In AVNRT, for example, while
while the child is in regular sinus, a premature
atrial beat may block in the fast pathway (because of its longer refractory period). Since the fast pathway is
blocked, the signal conducts down the slow pathway. Then, when the impulse reaches the insertion of the fast
pathway, which has now recovered after its refractory period, the impulse conducts in a retrograde fashion
through the fast pathway. This results in a circuit loop tachycardia, with an impulse moving in a loop down the
slow pathway and up the fast one.
Other causes of PSVT include sympathomimetic stimulation (such as medications for upper respiratory
infection), structural defects, and atrial ectopy. About half of all SVT cases occur without underlying heart
disease; these cases are termed idiopathic.
idiopathic. Idiopathic SVT is more common in younger patients than in older

3o clinical cases from eMedicine


children. WPW syndrome preexcita on accounts for 10-20% of cases. Congenital heart defects may also
predispose children to SVT. Children who have undergone cardiac surgery are also more prone to developing
this arrhythmia.
The diagnosis of PSVT is made based on patient history, physical examination findings, and EKG findings. The
EKG ndings include an excessively rapid (usually between 200 and 280 bpm) regular tachycardia, o en without
discernible P-waves preceding the QRS complexes. P-waves, when visible in PSVT, may have an abnormal axis
and may be seen within or following the QRS complexes. In most cases, the tachycardia will be narrow complex;
however, wide-complex PSVT is possible if aberrancy is present or if the AVNR conduction occurs in an
antidromic fashion with WPW syndrome. It is important to distinguish PSVT from sinus tachycardia, which is the
most common tachycardia in children. Sinus tachycardia and PSVT have dramatically different treatments.
Laboratory evaluation will depend on the clinical scenario, but practitioners should consider checking
electrolytes, thyroid function, and hemoglobin level.
Emergency management of children with PSVT should start with a focus on first assessing the airway for
potential compromise or obstruction, then evaluating breathing and ventilation, and assessing the circulatory
status of the patient (ie, the "ABCs"). When evaluating the circulatory status, unstable patients should undergo
immediate synchronized cardioversion with 0.5 J/kg. The power should be increased as needed to 2 J/kg. If the
cardioversion fails, overdrive pacing is another option. For stable patients, vagal maneuvers can be attempted
first. Infants possess a diving reflex, in which vagal tone will increase in response to a cold stimulus (eg, ice) on
the face. In older children, unilateral carotid massage, eyeball pressure, or even a headstand is more likely to
cause conversion. When these maneuvers are not typically successful in converting the rhythm, medication is
usually required. Adenosine, given at a dose of 0.1 mg/kg rapid intravenous push, is the rst-line agent. This
may be repeated in doses of 0.3 mg/kg, as needed. Amiodarone and procainamide are also safe in children with
SVTs refractory to adenosine. In general, adenosine and digoxin should be avoided in situations where there is a
suspicion of WPW syndrome, as they may potentiate conduction through the accessory pathway, resulting in
increased ventricular rates and possible degeneration to ventricular fibrillation. Children without WPW
syndrome preexcita on can be maintained on chronic oral propranolol or verapamil (in children older than 5
years), if needed after the acute illness. These agents should be used in consultation with a pediatric
cardiologist.
Disposition of children with PSVT depends on the clinical circumstance. A child in shock or with concerning
comorbidities should be admitted to a pediatric intensive care unit (ICU). Asymptomatic children without
frequent recurrence may be able to follow up with a cardiologist, without initiating medications. For younger
infants, the admission threshold should be lower.
The patient in this case failed cardioversion with vagal maneuvers but converted after two boluses of
adenosine, which is consistent with a reentrant circuit involving the AV node. The patient was admitted to a
monitored pediatric unit and evaluated by pediatric cardiology. The patient was initially discharged on digoxin;
however, after further follow-up, the medication was discontinued, because the child's cardiologist came to
suspect WPW syndrome as the likely etiology for the PSVT.

3o clinical cases from eMedicine


Case 27
INSIDIOUS SWELLING IN THE NECK OF A 45-YEAR-OLD MAN
Background
45-year-old man presents to his primary care physician (PCP) complaining of gradual swelling in his anterior
neck over the past 6 months. At rst, he thought nothing of the swelling, expec ng it go away on its own;
e concerned that the
however, over the past 2 months, it has become more no ceable. The pa ent has becom
swelling may be caused by cancer. He has not experienced any pain in the area of the swelling, nor has he
experienced fevers. Additionally, he denies any difficulty in swallowing and any alteration of his voice. There are
no problems with his breathing. The patient has no history of trauma, and he denies any significant personal
medical history. The family history is unremarkable. He is not currently taking any medications, he does not
smoke, and he does not use any illicit substances. He is a social drinker.

Figure 1

On physical examina on, the pa ent is afebrile and has a pulse of 72 bpm, a blood pressure of 130/82 mm Hg, a
respiratory rate of 12 breaths/min, and a normal oxygen satura on while breathing room air.He is welldeveloped and well-appearing. Examination of the anterior neck reveals a nontender, nonerythematous,
uctuant mass measuring approximately 10 8 cm in the midline of the lower neck, with slight extension to the
right side of the midline. The mass moves up and down when the patient swallows, and it slightly displaces
anteriorly with protrusion of the tongue. No cervical lymphadenopathy is appreciated. The lung fields are clear
bilaterally, without any evidence of stridor or wheeze. The heart has a regular rate and rhythm, without
murmurs, and the abdomen is soft and nontender, without evidence of masses. The cranial nerves are intact,
and the remainder of the neurologic exam is unrevealing as well.

3o clinical cases from eMedicine


Some routine laboratory blood tests, which include a complete blood cell (CBC) count and an electrolyte panel,
are sent, as well as a rapid assay for thyroid function tests. All of the laboratory investigations, including the
thyroid studies, are within normal limits. An ultrasound of the neck is obtained (Figure 1). As a follow-up, a
computed tomography (CT) scan of the neck is also performed for further evalua on (Figure 2).

Figure 2

What is the diagnosis?


Hint: This is the most common congenital anomaly resulting in a midline neck mass.
o
o
o
o

Cystic vascular abnormality


Cervical teratoma
Ectopic thyroid
Thyroglossal duct cyst

3oo clinical cases from eMedicine


Case 27 Answer
INSIDIOUS SWELLING IN THE NECK OF A 45-Y
45 EAR-OLD MAN

Diagnosis: Thyroglossal duct cyst


Discussion:
The ultrasound image in Figure 1 shows a large cys c mass anterior to the thyroid gland (arrowheads). The
contrast-enhanced
enhanced CT scan (Figure 2) demonstrates that same predominantly midline cystic mass extending
anteriorly to the thyroid gland and under the strap muscles, without any evidence of ectopic thyroid tissue. The
findings are consistent with the diagnosis of a thyroglossal duct cyst. This diagnosis is the most common
etiology for a midline neck mass. Thyroglossal duct cysts usually occur between the hyoid bone and the thyroid
gland, and they represent up to 70% of congenital neck anomalies. Thyroglossal duct cysts are second only to
lymphadenopathy as the most
ost common cause of a neck mass.
The cysts usually appear in the midline and can be present anywhere along the line of fetal descent from the
foramen cecum to the level of the thyroid gland. From an embryologic perspective, the thyroid gland develops
duringg the third week of life as an outgrowth of the floor of the primitive pharynx. The primitive thyroid then
descends from the foramen cecum to its mature position in the anterior neck through the thyroglossal duct. The
thyroglossal duct is normally resorbed by 7 to 10 weeks of fetal life. Abnormal persistence of the thyroglossal
tract accompanied by mucus production from the endothelial lining of the tract leads to the development of a
thyroglossal duct cyst. Approximately 7% of the popula on has thyrogloss
al duct remnants, and the distribution
is equal among males and females. The cysts are usually found in children or adults younger than age 30 years,
but they can develop in adults of any age. In recent years, a number of older patients, including patients in their
80s and 90s, have presented with thyroglossal duct cysts. There are 4 general types of thyroglossal duct cysts:
thyrohyoid (61% of cases), suprahyoid (24%), suprasternal (13%), and intralingual (2%).
The differential diagnosis for neck masses can be categorized by the location of the mass itself; the usual
categorization is between lateral and midline masses. The most frequent causes of lateral masses are
lymphadenopathy, branchial cleft cyst malignancy, cystic lymphangioma, and dermoid and terato
teratoid cysts.
Although thyroglossal duct cysts are the most common etiology for midline masses, the differential diagnosis
also includes dermoid and teratoid cysts, ectopic thyroid tissue, malignancy, and cystic lymphangiomas.
On radiologic images, thyroglossall duct cysts appear as a cystlike mass along the course of the thyroglossal duct.
They must be differentiated from dermoid cysts and lymphangiomas. A dermoid cyst usually contains fat;
lymphangioma is most common in infancy or early childhood, and it usually
usually occurs in the posterior triangle of
the neck, behind the sternocleidomastoid muscle. A thyroglossal duct cyst must also be differentiated from an
ectopic thyroid gland. If an ectopic thyroid gland is mistakenly removed, the patient may require long
long-term
thyroid treatment for hypothyroidism. Often, patients with an ectopic thyroid also have hypothyroidism, and
they will have an elevated thyroid-stimulating
stimulating hormone (TSH) level.
The diagnosis of thyroglossal duct cyst is made on the basis of the clinical history and confirmed with diagnostic
imaging. Most patients with thyroglossal duct cysts present with either a history of a slowly growing,
asymptomatic mass or of a relatively rapid-growing
r
growing mass (if the cyst is infected) in the anterior midline of the
neck. Frequently, the swelling is exacerbated during an upper respiratory infection. The pathognomonic sign of

3o clinical cases from eMedicine


a thyroglossal duct cyst is that it moves with swallowing and with protrusion of the tongue; however, the
mobility of larger cysts may be restricted.
Imaging studies, including ultrasonography and CT scanning of the neck, will confirm the diagnosis and help to
rule out the presence of a possible ectopic thyroid gland. Thyroid function tests should be obtained to confirm
normal thyroid function.
Once diagnosed, thyroglossal duct cysts are removed because they are cosmetically undesirable and have the
potential to become infected and undergo malignant transformation. The treatment of choice is the Sistrunk
procedure, named a er Dr. Walter Ellis Sistrunk and rst described in an ar cle in 1920. The procedure, rather
than involving a simple excision of the cyst, involves dissecting the central portion of the hyoid bone, with
extension of the excision up to the base of the tongue to include excision of a small block of muscle around the
foramen cecum. Because of the increased risk of thyroglossal duct carcinoma, some practitioners further
recommend the addition of thyroid suppression therapy or a complete thyroidectomy; however, this practice
remains controversial. The recurrence rate associated with simple excision of a thyroglossal duct cyst is
approximately 50%, whereas the recurrence rate associated with a formal Sistrunk procedure is approximately
5%. The rate of recurrence a er a Sistrunk procedure is increased, however, when a thyroglossal duct is
ruptured during dissection. A history of previous infection of the cyst, previous incision and drainage
procedures, and adherence of the cyst to the skin all increase the risk of rupture during dissection. If the cyst is
infected at the time of diagnosis, treatment with antibiotics, such as ampicillin/sulbactam,
amoxicillin/clavulanate, or clindamycin, is indicated before surgical excision.
The most common complications of thyroglossal duct cysts are infection with the possibility for abscess
formation, spontaneous rupture, and formation of a secondary sinus tract. A Sistrunk procedure mistakenly
performed for thyroid ectopia that removes thyroid tissue can cause hypothyroidism. The cysts can compress
the trachea and lead to respiratory distress, especially if they are rapidly expanding (although this is not
common). Carcinoma is the most feared complica on, occurring in about 1% ofall cases, with papillary
carcinoma accoun ng for 85-92% of malignancies and follicular carcinoma accoun ng for the rest. Most
patients who develop carcinoma tend to present at a later age. Cancer in a thyroglossal duct cyst seems to be
more common in females than in males. The diagnosis of carcinoma arising in a thyroglossal duct cyst is
typically made postoperatively by histology.
Because the patient in our case presented with a thyroglossal duct cyst which (on clinical grounds) was not
infected, he was not given antibiotics. The ultrasound study confirmed the presence of a thyroglossal duct cyst
and ruled out the possibility of an ectopic thyroid gland. The patient underwent an elective Sistrunk procedure,
with no rupture during dissection. There were no complications. He was discharged from the hospital the
following day. Postoperative histologic analysis did not reveal any evidence of malignancy. On follow-up in the
clinic 2 weeks later, the pa ent was noted to be doing well.

3o clinical cases from eMedicine


Case 28
NEAR-SYNCOPE IN A 24-YEAR-OLD MAN
Background
A 24-year-old man with no significant past medical history presents to the emergency department (ED) with a
complaint of several episodes of a sensa on of nearly blacking out. The episodes have occurred about 3-4 mes
over the 3 days before presenta on. The dura on of each episode has ranged from a few minutes to over an
hour. The patient notes that he has felt his "heart beating really fast," with associated light-headedness. He
denies having any chest pain, shortness of breath, or nausea associated with these events. He cannot identify
exacerbating or alleviating factors; specifically, he denies exertion as an inciting factor. The remainder of his
review of systems is negative except for some mild chronic shortness of breath. The patient takes no
medica ons at home and has no ac ve medical condi ons. He smokes
-4 2packs of cigare es per day and has
done so for 5-6 years. He denies any illicit drug use or recent use of over-the-counter medications or herbal
remedies. He has no history of any significant cardiac disease or sudden cardiac death in his family.

Figure 1

On physical examina on, the pa ent is afebrile, with a pulse of 65 bpm, a blood pressure of 120/84 mm Hg, and
a respiratory rate of 15 breaths/min. His room air satura on reading is 100%. In general, he is well-appearing
and in no acute distress. The patient's neck examination shows no jugular venous distention. The heart sounds,
including S1and S2, reveal no audible murmurs, rubs, or gallops. The apical impulse is nondisplaced and of
normal impact. The lung sounds are diminished throughout, but there are no wheezes, rales, or rhonchi. He has
no edema of the lower extremities, and the distal pulses are easily palpable. All other exam findings, including a
neurologic examination, are unremarkable.
The pa ent is placed on a cardiac monitor, and an 18-gauge intravenous (IV) catheter is inserted into the
antecubital fossa. Laboratory tests consisting of a complete blood count (CBC) and serum electrolytes are
ordered. A portable chest radiograph reveals slight hyperinflation and hyperlucency of the lung fields, with a
flattened diaphragm and central pulmonary artery enlargement. An electrocardiogram (ECG) is obtained (see
Figure 1).

3o clinical cases from eMedicine


What is the diagnosis?
Hint: Pay close attention to the intervals and the QRS complex morphology.
o
o
o
o

Wolff-Parkinson-White syndrome
Ventricular fibrillation
Sinus tachycardia
Non-sustained ventricular tachycardia

3oo clinical cases from eMedicine


Case 28 Answer
NEAR -SYNCOPE IN A 24-YEAR -OLD MAN

Diagnosis: Wolff-Parkinson-White
White syndrome
Discussion:
Preexcitation is characterized by an accessory pathway within the heart that conducts action potentials
between the atria and ventricles outside of the normal conduction system (which conducts through the
atrioventricular [AV] node-His-Purkinje
Purkinje system). The phenomenon was dened by Durrer et al in 1970, who
stated that "preexcitation exists, if in relation to atrial events, the whole or some part of the ventricular muscle
is activated earlier by the impulse
pulse originating from the atrium than would be expected if the impulse reached
the ventricles by way of the normal specific conduction system only. Of the various types of preexcitation
syndromes, the most common is Wolff-Parkinson-White
Wolff
(WPW) syndrome.

Figure 1

WPW syndrome can be identified by a classic fusion QRS complex ECG pattern that is a combination of
simultaneous normal conduction through the AV node and aberrant conduction through the accessory tract.
This fusion QRS complex leads to par cular ECG features that include a shortened PR interval (<120 msec) and a
widened QRS complex with a delta wave representing preexcitation of the ventricle through the accessory
pathway. The distinctive ECG pattern of the accessory pathway was initially
initially described by Wolff, Parkinson, and
White in 1930 as a bundle branch block with a short PR interval. Addi onally, as men oned, WPW syndrome is
recognized as the most common form of ventricular preexcitation, although it likely represents a collecti
collection of
pathologic conditions rather than a single structural abnormality.

3o clinical cases from eMedicine


Normal cardiac conduction of action potentials from the atria to the ventricles occurs exclusively through the
AV node; the atrial impulses are subsequently propagated through a specialized conduction system (the AV-HisPurkinje system) and finally terminate in the ventricular myocardium. Action potential conduction through the
AV node depends on slow inward calcium currents. In addition, the AV nodal system exhibits decremental
conduction, which provides a protective effect; as the cardiac cycle is shortened (eg, the heart rate increases),
there is decreased conduction through the AV node. This phenomenon limits the ventricular response to rapid
atrial rates, such as those observed in atrial fibrillation or atrial flutter.
In preexcitation syndromes such as WPW, however, the action potential conducts to the ventricles at least
partially through an accessory pathway termed the AV bypass tract or the bundle of Kent. Action potential
propagation in the accessory pathway in WPW syndrome occurs through a rapid cellular influx of sodium. The
consequence of the sodium-dependent action potential propagation mechanism is an accelerated conduction
of impulses by the accessory bypass tracts, which leads to early activation of the ventricle as demonstrated by a
shortened PR interval and a "slurred" QRS complex (ie, delta wave). Ventricular depolarization slowly spreads
out from the bypass tract, while normal conduction that has been somewhat delayed through the AV node
begins to conduct through the His-Purkinje system and spreads quickly to the remaining ventricular
musculature. Although conduction velocity through the accessory pathway is faster than it is through the AV
node, the accessory pathway often has a longer refractory period and, as such, is slower to recover excitability.
Interestingly, the conduction of action potentials through the accessory pathway is nondecremental; therefore,
the protective effect achieved by the AV node at higher heart rates is lost. These differences have important
clinical implications. For example, a premature beat may conduct through the AV node normally while the
accessory pathway remains refractory to conduction. The impulse then travels in a retrograde direction through
the accessory pathway after ventricular depolarization, when it has recovered excitability. The consequence of
this is the propagation of a reentry loop termed an orthodromic AV reciprocating tachycardia. This can then
lead to rapid ventricular response rates that can degenerate into ventricular tachyarrhythmias. Rarely,
antidromic tachycardias occur; conduction occurs in an anterograde direction through the accessory pathway
and in a retrograde direction through the AV node.
Ventricular depolarization occurs through both the AV node-His bundle pathway and the accessory pathway;
each pathway affects the ventricles by various degrees, depending on their relative activation times. As AV
nodal conduction is delayed by either rapid atrial pacing or premature atrial beats, the accessory pathway
contributes to a greater degree, resulting in a wider QRS morphology with an increasingly slurred delta wave. If
the relative conduction time through the AV node is sufficiently delayed, total activation of the ventricle may
occur through the accessory pathway.
The presence of accessory bypass tracts is not uncommon in the general population; however, less than half of
the people with bypass tracts actually sustain a tachyarrhythmia. WPW syndrome affects approximately 0.150.2% of the general popula on, and of these individuals, 60-70% have no other evidence of heart disease.
Mortality and morbidity associated with WPW syndrome occur as a result of associated dysrhythmias or from
mistreatment of these dysrhythmias with inappropriate medications. Most studies report that the incidence of
sudden death is in the 0-4% range. Men are aected more o en than women, accoun ng for 60
-70% of all
cases. Although this disease affects people of all ages, it is typically first recognized in children and young adults
who present to the ED or their primary care physician with symptoms secondary to a dysrhythmia. Genetic
mutations have been identified (by mapping genetic defects to specific loci) that account for the increased
incidence of WPW syndrome in certain families.
In patients with suspected WPW syndrome, evaluation should initially be directed at confirming the diagnosis
and recognizing any potentially life-threatening arrhythmias. In patients with life-threatening arrhythmias,
direct-current cardioversion should be immediately administered. In stable patients with tachyarrhythmias, an
antiarrhythmic medication may be administered to terminate the arrhythmic episode, rather than immediately
performing electrical cardioversion.
Studies have demonstrated that the best and most cost-effective treatment for patients with asymptomatic
WPW syndrome is simple observation. Most patients with symptomatic arrhythmias, drug-refractory WPW
syndrome, or significant life-threatening arrhythmias are treated with nonpharmacologic therapy.

3o clinical cases from eMedicine


Surgical ablation, previously the standard technique for drug-refractory WPW syndrome, has been replaced by
catheter-based procedures. Compared with surgical techniques, catheter ablation has comparable success
rates, lower mortality and complication rates, and improved cost-effectiveness. Moreover, newer catheter
mapping systems now allow shorter procedure times. Surgical ablation, however, may be necessary in patients
in whom catheter ablation has failed. Because this patient had a symptomatic tachyarrhythmia, he underwent
electrophysiologic mapping followed by transvenous catheter ablation. He has remained asymptomatic since
this procedure.
Patients with infrequent or minimally symptomatic arrhythmias may be treated pharmacologically. The aim of
pharmacologic therapy is to alter the electrophysiologic properties, such as the refractoriness or conduction
velocity of the AV node or the accessory bypass tracts.

3o clinical cases from eMedicine


Case 29
NONRESOLVING PNEUMONIA IN A YOUNG WOMAN
Background
A 33-year-old woman was evaluated because of persistent pulmonary inltrates. She was well un l 7 months
ago, when she developed fever, a productive cough, and left-sided pleuritic chest pain. A chest radiograph
showed an infiltrate in the left lower lobe. Several courses of antibiotics led to an improvement of her
symptoms, though a dry cough persisted. Subsequent chest CT showed attenuating airspace disease in the left
lower lobe with air bronchograms and ill-defined nodules in both lungs. Before her current presentation, she
underwent 2 bronchoscopic procedures, with transbronchial biopsy performed during the most recent one. All
bacterial cultures were negative, and cytology and pathology failed to reveal malignancy. Given the lack of an
infectious etiology, corticosteroids were started. However, the patient's condition did not improve.

Figure 2

On evaluation in the emergency department, the patient was asymptomatic except for a persistent cough with
clear sputum. She was a nonsmoker and had no previous pulmonary disease. She did not have a history of
tuberculosis or known exposure to risk factors. A purified protein derivative of tuberculin (PPD) test performed
3 months ago yielded nega ve resul
ts. She has not been taking any drugs and has no known allergies. Physical
examination revealed a well-appearing woman in no distress with a respiratory rate of 16 breaths per minute, a
temperature of 35.9C, a blood pressure of 110/60 mm Hg, and a heart rate of 95 beats per minute. Her lungs
were clear, with no wheezing, rhonchi, or rales. Her heart sounds were normal, with no murmurs. The
remainder of her examination yielded unremarkable findings.

3o clinical cases from eMedicine

Figure 3

Chest CT was performed (see Image). Positron emission tomography (PET) showed increased uptake in the left
posterior por on of the lower lung with a standard uptake value (SUV) of 6.5 and no uptake in the nodules or
hilar or mediastinal nodes.

What is the diagnosis?


Hint
The patient's indolent presentation is typical of this condition.

3o clinical cases from eMedicine


Case 29 Answer
NONRESOLVING PNEUMONIA IN A YOUNG WOMAN
Diagnosis: Bronchoalveolar carcinoma (BAC)
Discussion:
Biopsies of the left lower and upper lobes of the lung were performed by means of video-assisted thoracoscopic
surgery (VATS). The results indicated a stage IV BAC.
Despite serial chemotherapy over 4 years, the disease progressed (see Image 2). The pulmonary inltrates
expanded and became cavitated. The patient's respiratory status worsened, and she became oxygen dependent
at rest. Recurrent pulmonary infections and airway obstruction secondary to tumor progression resulted in
progressive respiratory failure, which ultimately led to her death.
Lung cancer is the most common cancer worldwide and has the greatest mortality rate. Lung adenocarcinoma is
the most common histologic form of lung cancer (31.5%). Its frequency is rising in women and in nonsmokers.
BAC is a subtype of lung adenocarcinoma and has a wide variety of distinct clinical manifestations, which often
lead to an ini al misdiagnosis. It is rela vely rare (accoun ng for 2.6% of all the lung cancers), and it is seen
mainly in women aged 40-70 years. It typically has a rela vely indolent course, an
d its association with smoking
is weaker than that of other forms of lung cancer.
In 1999, the World Health Organiza on (WHO)International Association for the Study of Lung Cancer (IASLC)
classification defined BAC as an adenocarcinoma with a lepidic growth pattern (growth along intact alveolar
septa) without invasive growth, such that the alveolar architecture remains intact. Patients with BAC may
present with one or several nodules or with diuse parenchymal inltrates. More than 50% of BACs may be
associated with focal scars. A final diagnosis of BAC can be achieved only by analyzing a surgical specimen.
Metastatic adenocarcinomas can mimic BACs; therefore, a history of an extrathoracic adenocarcinoma
precludes the diagnosis of BAC.
The 2 main histologic types of BAC are mucinous and nonmucinous. The mucinous type (41-60%) has a colloidlike histologic appearance because of mucin that fills the adjacent alveoli. In clinical terms, this finding manifests
as the expectoration of mucoid material. The mucinous form tends to be multicentric and often causes lobar
consolidation resembling that of pneumonia, and it worsens the patient's prognosis. Whether the
multicentricity is due to aerogenous or lymphatic spread or the expression of several independent malignant
clones is still debated.
The nonmucinous type (21-45%) is composed of cuboidal or columnar cells. It is most likely to result in a solitary
nodule and improves the prognosis. Approximately 12-14% of BACs are of a mixed subtype, and as many as 7%
are indeterminate.
About 50-60% of pa ents are asymptoma c in the early stages, and the diagnosis may be made incidentally
during chest radiography. Cough (35%), sputum (24%), and shortness of breath (15%) are usually seen in only
advanced disease. Weight loss, hemoptysis, and fever occur less frequently than these other findings.
Bronchorrhea (5%) is a late sign and most o en associated with the diuse, mucinous form. Diuse lling of the
alveolar spaces by mucin causes severe hypoxia, which is also a manifestation of late disease.
The radiologic presenta on of BAC varies, ranging from solitary (43%) or mul ple (27%) nodules to diuse
airspace disease (30%). The CT a enua on of these forms ranges from that of so
ssue (frank inltrate) to
ground-glass opacity (hazy shadow). A solitary pulmonary nodule is typically peripheral, slowly growing, and
unlikely to evolve to widespread disease. Airspace disease (pneumonic form) can be indistinguishable from
pneumonia and is due to a combination of tumor cells and mucin. Multiple nodules are often bilateral and
sometimes mimic pulmonary metastatic disease. In rare cases, BAC can manifest as lobar atelectasis or as cysts
and cavi es (7%).

3o clinical cases from eMedicine


Certain radiologic signs should raise a clinician's suspicion for BAC: air bronchograms (open bronchus in an area
of infiltrate), pseudocavitation (low-attenuation regions in a nodule), or a CT angiographic sign (visualization of
vessels in regions of consolidation after intravenous administration of contrast material).
PET is often used to evaluate BAC. BAC usually has an SUV lower than that of other lung cancer subtypes; the
decreased value is due to low metabolic activity or low cellularity. In general, negative PET results are
associated with focal disease, and increased uptake is associated with diffuse and poorly differentiated forms.
The differential diagnosis of the pneumonic form of BAC includes all the infectious causes of nonresolving
pneumonia, other primary or metastatic lung malignancies, lymphoma, sarcoidosis, pulmonary alveolar
proteinosis, diffuse alveolar hemorrhage, vasculitis, eosinophilic pneumonia, bronchiolitis obliterans organizing
pneumonia (BOOP), or pulmonary edema.
The mucinous type, diffuse lesions, and symptomatic disease, are associated with a worsened prognosis.
Pa ents with disease in the T1N0M0 stage have a 90.5% 5-year survival rate, as opposed to those with diffuse
bilateral disease, who have 0% survival at 5 years. A 100% 5-year survival rate is reported for BAC with a >75%
lepidic growth pa ern and a central scar <5 mm or an intact elas -fiber
c framework.
No optimal therapy has been defined for recurrent or advanced BAC. The response rate to conventional
chemotherapy is low (14% response rate to a 96-h infusion of paclitaxel). Recent studies showed improved
results with epidermal growth factor (EGFR) or tyrosine kinase inhibitor. Women without a history of smoking
appear to respond most o en to ge nib. The reported response rate is 12
-19% for ge nib and 26% for
erlotinib. Somatic mutations of EGFR appear to make the BAC responsive to these targeted inhibitors in
approximately only 10% of pa ents. Surgical op ons include cura ve resec on for limited forms (usually up to 3
separate nodules), palliative resection, or, potentially, lung transplantation.
This case involved a rare presentation of the pneumonic form of BAC in a young patient with a poor response to
treatment. BAC has clinical, histologic, and radiographic characteristics distinct from those of other nonsmall
cell lung cancers. Being familiar with the multiple forms of presentation of BAC can help in the early diagnosis
and subsequent treatment of a potentially curable disease.

3o clinical cases from eMedicine


Case 30
RESUSCITATION FROM VENTRICULAR FIBRILLATION ARREST
Background
The patient was resuscitated from cardiac arrest caused by ventricular fibrillation. What is the diagnosis?

Figure 4

Hint
Check out those ST segments!

3o clinical cases from eMedicine


Case 30 Answer
RESUSCITATION FROM VENTRICULAR FIBRILLATION ARREST
Diagnosis: Acute anterior-wall myocardial infarction
Discussion:
Note the hyperacute T waves in the anterior leads and the elevated ST segments in these leads. Although the
eleva ons are not convex in many leads, they are in leads V1 and aVL. Furthermore, the reciprocal ST-segment
depressions in the inferior leads confirm the diagnosis.

You might also like